Söker efter kvarkar

 

123 frågor/svar hittade

Partiklar [119]

Fråga:
Vad är en kvark? Hur är den uppbyggd?
/

Svar:
Kvarkar är punktformiga partiklar som tre och tre har slagit sig samman och bygger upp neutroner eller protoner. Uppkvarken har laddningen + 2/3 och nedkvarken laddningen -1/3, uttryckt i elementarladdningen som enhet.

Upp- och nedkvarken är, tillsammans med elektronen, de minsta nu kända partiklar som bygger upp den "normala" materien. Protonen består av två uppkvarkar och en nedkvark medan neutronen innehåller två ned- och en uppkvark.

Subkvarkar skulle vara partiklar som i sin tur bygger upp kvarkarna. Några sådana partiklar har man inte funnit.

Läs: Forskning och Framsteg har de senaste åren haft flera bra artiklar i detta ämnesområde.

*

Partiklar [120]

Fråga:
Hur påvisades kvarkarna, och vad finns det för olika typer?
/

Svar:
Kvarkarna upptäcktes genom att man sköt elektroner med mycket hög energi in i protoner. Man fann då att de flesta gick rakt fram medan några studsade rakt bakåt. Detta tolkade man som om protonen bestod av små hårda partiklar som kallades för kvarkar. Jämför med Rutherfords guldfolieförsök.

För att studera kvarkar använder man stora partikelacceleratorer. När man upptäckte kvarkar för första gången så användes en elektronaccelerator. Man måste ha väldigt hög energi eftersom deBroglie-våglängden hos elektronerna måste vara mindre än avståndet mellan kvarkarna inne i protonen (neutronen).

Beräkna: Protonen har en radie på ca 1 fm. Vilken energi måste elektronen minst ha?

I protoner och neutroner finns u och d kvarkarna. Man har även lyckas hitta andra kvarkar i kortlivade partiklar. Dessa kallas för s, c, t och b.

u står för upp, d (ner, "down"), s (sär), c (charm), t(topp) och b (botten).

Läs: Nobelpriset i fysik 1990 gick till upptäckten av kvarkar, se The Nobel Prize in Physics - Laureates . I KOSMOS 1991 finns en beskrivning av den forskning som ledde till detta nobelpris.

*

Partiklar [121]

Fråga:
Jag håller på att läsa lite om kvarkar, och har där åtskilliga gånger snubblat över symmetrigruppen SU(3) utan att man angivit vad denna innebär. Kan ni möjligtvis ge ett svar?
/

Svar:
Det är inte lätt att förklara vad den abstrakta gruppen SU(3) är för något med hjälp av enbart gymnasiematematik. Man kan säga att SU(3) håller ordning på vilka olika tillstånd som tre kvarkar (u, d och s) kan finnas i . Du har säkert sett figurer som trianglar och sexhörningar med olika partiklar i hörnen. Med hjälp av SU(3) kan man bestämma de olika former på dessa figurer som kan finnas.

*

Partiklar [123]

Fråga:
Vi upptäckte häromdagen i standardmodellen att lambda-0 och sigma-0 har samma kvarkuppsättning (uds). Hur är detta möjligt? Vad skiljer dem åt?
/

Svar:
Du har rätt, båda partiklarna är uppbyggda av samma kvarkar. Vad som skiljer dem åt är den inre strukturen. Bl a har kvarkarna precis som elektronen spinn. I sigma-0 har u och d-kvarkarna parallella spinn medan dessa två kvarkar har antiparallella spinn i lambda-0. Skillnaden i inre struktur avspeglar sig sedan i partiklarnas egenskaper som t ex massa.

I partikelfysiken har man en lite annorlunda terminologi än i atom- och kärnfysik. Man säger t ex inte att ett exciterad tillstånd i väteatomen är en ny atom. I partikelfysiken däremot så får det exciterade tillståndet status som en egen partikel.

*

Blandat [402]

Fråga:
Vad är fysik?
vem kom på det?
varför?
när?
var?
vad har vi för nytta med det?
/   

Svar:
Fysik är den mest grundläggande av naturvetenskaperna och behandlar allt från det största i universum (stjärnor, galaxer, hela universum) till de minsta partiklarna (elektroner, protoner, kvarkar osv). Det handlar också hur de små partiklarna bildar olika former av materia (atomkärnor, atomer, molekyler, fasta kroppar) och hur dessa kroppar påverkar varandra och hur de kan röra sig.

Det var nog först på 1500-talet som fysik i vår mening började. Ska vi nämna ett namn så får det bli Galileo Galilei.

Människan är nyfiken och vill veta hur världen ser ut och hur den fungerar. Kanske är fysikens viktigaste område just nu energifrågorna. Fysiker i hela världen forskar kring hur man ska kunna utvinna energi så billigt och miljövänligt som möjligt. Hur de forskningsresultaten ska komma till nytta är lätt att förstå. Men det är inte all vår forskning som känns lika självklart användbar idag. Vi är dock övertygade om att den kunskap vi bygger upp behövs och kommer att behövas för att lösa problem vi kommer att möta i framtiden.

Utan fysik och naturvetenskap överhuvudtaget så hade vi inte kunnat ha någon teknologisk utveckling. Den förste att utnyttja fysik var den som lärde sig att tända eld. Hade inte männinskan försökt att förstå och utnyttja naturen så hade vi fortfarande bott i grottor och haft en medellivslängd på under 20 år.

Tänk: Vilka apparater skulle vi haft om vi inte haft elektricitet?

Läs: Tycker Du fysik verkar roligt ska Du läsa boken "Påsen och sorken gör experiment" av Hans-Uno Bengtsson.

Se även fråga 12584

Nyckelord: fysik, nytta med [6];

*

Universum-Solen-Planeterna [469]

Fråga:
Vad är kvark-gluon-plasma??
/

Svar:
Mycket tidigt i universums historia (universums ålder var mindre än en hundratusendels sekund) bestod materien väsentligen av en ³soppa³ av kvarkar och gluoner. Det är detta som kalls för kvark-gluon-plasmat. I dagens högenergiacceleratorer så försöker man återskapa små bitar av detta plasma genom att slå ihop två atomkärnor med mycket hög energi.

Läs Uppslagsordet universum i Nationalencyklopedin ger en mycket fin beskrivning av universums utveckling.

1 http://www.kosufy.lu.se/link/smain.html

*

Partiklar [520]

Fråga:
Hej. Min fråga är om den minsta byggstenen, kvarken tror jag den heter. Jag har för mig att jag sett på tv att de har lyckats att fotografera dessa små ting men min lärare i religion påstår motsatsen, alltså frågan är: Har vetenskapen lyckats med att fotografera de minsta byggstenarna i en atom eller inte?
/

Svar:
Nej! Man har lyckats fotografera enstaka atomer men kvarkar är minst 100 000 gånger mindre. Det torde vara helt omöjligt att fotografera dessa partiklar med apparater som till sin konstruktion påminner om mikroskop. Tyvärr har din religionslärare rätt!

*

Partiklar [521]

Fråga:
Vad är elementarpartiklar uppbyggda av??
/

Svar:
Partiklar som fotonen, elektronen, neutrinon är inte uppbyggda av några mindre beståndsdelar. Man anser att de är punktformiga och har egenskaper som (vilo-) massa, laddning etc. Däremot är neutronen och protonen uppbyggda av mindre partiklar som kallas för kvarkar.

*

Partiklar [607]

Fråga:
Varför är massan på 3 up eller nerkvarkar så otroligt mycket mindre än protonens massa.


/   

Svar:
Vad menas med massan hos en kvark? Det är ingen trivial fråga eftersom kvarkarna alltid är bundna tillsammans, som t ex i protonen, vilka består av tre kvarkar. Men det finns inte bara kvarkar inne i protonen utan även ett "moln" av s k virtuella gluoner. Detta moln omger kvarkarna. Man säger ofta att kvarkarna är klädda. En sådan "påklädd" kvark har en massa som är ca 1/3 av protonens massa, vilket i energienheter blir ca 300 MeV.

Om man däremot skjuter in en elektron med mycket hög energi i protonen så kan den träffa en kvark som far iväg utan sitt moln av gluoner Man har fått en "naken" kvark. Mäter man dess massa så blir den endast ca 5 MeV.

Kommentar Kvarken som "far iväg" kommer inte så långt att den uppträder isolerat. Det är endast för en kort stund den rör sig fritt.

Fundera Hur kan man mäta massan med ett stötförsök?


*

Materiens innersta-Atomer-Kärnor [721]

Fråga:
Protoner innehåller ju kvarkar, två upp- och en ner- ( 2/3 +2/3 -1/3 =3/3 =1 ) och blir på så sätt positiv. Är det teoretiskt möjligt att i framtiden skjuta bort nerkvarken och skjuta in en uppkvark till så att protonen skulle bli dubbelt så positiv ( 2/3 +2/3 +2/3 =6/3 =2 )? Om detta skulle vara möjligt, skulle vi då få helt andra grundämnen ex. grundämnen med dubbelt så många elektroner som protoner?
/Henrik J, Park, Övik

Svar:
Partikeln har observerats, men den har mycket kort livstid. Den kallas D++. Det var i själva verket upptäckten av en mängd sådana här s.k. baryoner som gjorde att man hittade på kvarkteorin med de ovanliga laddningarna. Det är först mycket senare som man sett direkta bevis för tre partiklar som bygger opp neutroner och protoner. Eftersom D++-partikeln är instabil även inne i en kärna, så kan man inte bygga upp några nya grundämnen. Även om man kunde, så skulle man få samma grundämnen, eftersom deras egenskaper ges av laddningen i kärnan.
/LPE

*

Materiens innersta-Atomer-Kärnor [966]

Fråga:
Hallå på er alla på Resurscentrum! Vi har några frågor om energin till er.

1. Vad skulle hända om tråden i en glödlampa skulle vara rak i stället för snirklad?

2. Varför rör atomerna på sig?

3. Vad är neutriner får nåt?

4. Vad är subkvarkar?


/Andreas K, Tunaskolan, Lund

Svar:
1. Glödtråden går i "snirklar" därför att den behöver vara lång för att få rätt resistans så att den glöder lagom starkt. Skulle den vara rak så måste man göra den tunnare och då går den lätt av.

2. Därför att det är varmt. När man värmer en kropp rör sig atomerna allt snabbare. Om man kunde kyla till absoluta nollpunkten så skulle alla atomer ligga stilla bredvid varandra.

3. Neutriner är små masslösa partiklar som kan gå rakt igenom allting utan att stoppas. Genom våra kroppar flyger det miljarder neutriner från solen varje sekund.

4. Subkvarkar har man hittat på för att man tror att kvarkar är uppbyggda av ännu mindre partiklar. Det finns dock inget som tyder på att sådana skulle finnas.

 
/GO

*

Materiens innersta-Atomer-Kärnor [1290]

Fråga:
Har hört att man säger att det finns tolv grundpartiklar, men jag kan inte hitta någon uppställning som talar om vilka de är i så fall? Jag kan inte få ihop det på något sätt, jag tycker att det blir många fler även om man inte räknar med antipartiklarna. Fotonen, kvarkarna, de olika leptonerna och mesonerna borde vara med, men räknas även tex Higgs partikeln och liknande partiklar med?
/Gunnar O, Stenhamreskolan, Ljusdal

Svar:
Låt oss räkna de "riktigt" grundläggande partiklarna. Då tar vi inte med till exempel protonen, som består av tre kvarkar, och inte heller mesoner, som består av ett kvark-antikvarkpar.

Det finns 6 stycken kvarkar:
u (upp), d (ner), s (sär), c (charm), t (topp) och b (botten).

Det finns sex stycken leptoner:
elektronen och elektronneutrinon, myonen och dess neutrino samt tauonen och dess neutrino.

Dessa partiklar bygger upp all materia, både den som finns normalt och den som skapas i fysikens laboratorier.

Fundera Av vilka tre partiklar är all "normal" materia här på jorden uppbyggd?

Beräkna Kan Du räkna ut hur många kvarkar det finns i Din kropp?

Sedan finns det partiklar som är så kallade kraftförmedlare. De är:
Fotonen (elektromagnetisk kraft), W+, W- och Z0 (den svaga kraften) samt gluonen (den starka kraften).

Alla partiklarna som vi hittills räknat upp är påvisade experimentellt. Dessutom finns det två andra partiklar som många teorier förutsäger, nämligen Higgs-partikeln och gravitonen.

Precis som Du säger finns det också antipartiklar till alla partiklar.


/GO

*

Materiens innersta-Atomer-Kärnor [1357]

Fråga:
Jag har 6st frågor;

1, Jag har läst någonstans att gluoner är färgladdade, men inte får ha en neutral färgladdning. Jag har suttit & testat mig fram hur många varianter det kan finnas, men jag får det inte till det antal jag läst om. Jag har läst att det finns 8 varianter, men jag kan inte få till det. Kan ni hjälpa mig med det?

2, Finns det 3 färger, eller 6 färger (med antifärger), kan då vanlig materia ha en antifärg, eller finns det bara i antimateria?

3, Hur många dimensioner räknar fysiker med att det finns? & varför skulle en supersträng ha just 10 dimensioner?

4, I en teori för tiden, sägs imaginär tid kanske vara den universella tiden; hur kombinerar man detta begrepp med tiden?

5, Ifall subkvarkar finns, hur beter dom sig då enligt subkvarksteorin?

6, Vad är Tachyoner för överljushastighets partiklar?
/Hananja R, Grosvad, Finspång

Svar:
1 Gluoner kan finnas i varianterna av typ röd-antigrön. Det finns 6 sådana kombinationer. Dessutom finns det kombinationer av typen röd-antiröd - blå-antiblå som inte är färglösa trots att det verkar så. Enligt kvantmekaniken måste man lägga ihop färgladdningarna enligt vissa symmetrier och de kombinationer man får måste vara oberoende av varandra. Totalt blir det 8 stycken sådana oberoende tillstånd. Det går tyvärr inte att förklara med enkel matematik.

2 Kvarkar i vanlig materia har färg men inte antifärg. Däremot finns det antifärg i det "gluonmoln" som finns inne i nukleonerna. Detta märks inte utanför partikeln eftersom hela systemet (kvarkar + gluoner) är färgneutralt.

3 Supersträngar är rena tankekonstruktioner som rör sig i rum med en tidsdimension och flera rumsdimensioner. Man kan säga att man väljer det antal dimensioner som passar bäst och försöker sedan anpassa det till verkligheten som är fyrdimensionell (en tidsdimension och tre rumsdimensioner).

4 Imaginär tid är ett matematiskt knep som ibland underlättar beräkningar men som inte innebär någon förändring av det fysikaliska tidsbegreppet.

5 Det finns inga tecken som tyder på att det skulle existera subkvarkar.

6 Tachyoner finns endast i teoretiska tankelekar. Dessa partiklar skulle alltid röra sig fortare än ljuset. Det finns inga tecken som tyder på att sådana partiklar finns.
/GO

*

Partiklar [1424]

Fråga:
Hur kommer det sig att kvarkar och antikvarkar kan båda finnas i mesonerna? Varför förintas de inte som om materia möter antimateria?
/Veckans fråga

Ursprunglig fråga:
Hur kommer det sig att kvarkar och antikvarkar kan båda finnas i mesonerna? Varför förintas de inte som om materia möter antimateria?
/Henrik J, Parkskolan, Örnsköldsvik

Svar:
Det här är en mycket intressant fråga.

Jo, det är faktiskt så att kvarken och antikvarken i en meson annihilerar (förintar) varandra. Det sker när mesonen sönderfaller. Det finns ingen stabil meson.

Exempel 1, den neutrala pi-mesonen (har ingen elektrisk laddning)

Den neutrala pi-mesonen består av en kombination av [u och anti-u] och [d och anti-d]. För att kunna begripa detta fullt ut, måste man kunna ganska mycket kvantmekanik. I varje fall finns alla förutsättningar för annihilation, och det sker också nästan omedelbart med elektromagnetisk växelverkan. Den neutrala pi-mesonen är mycket kortlivad, 10-16 sekunder.

Exempel 2, den positiva pi-mesonen (har positiv elektrisk laddning)

Den positiva pi-mesonen består av [u och anti-d]. Här har vi två olika sorts kvarkar och de kan inte annihilera varandra direkt. En tredje partikel måste vara inblandad för att ta hand om den elektriska laddningen (som måste bevaras). Sönderfallet (kvark-annihileringen) sker genom den s.k. "svaga växelverkan" som, eftersom den är så svag, behöver lång tid på sig. Den positiva pi-mesonen lever 100 miljoner gånger längre än den neutrala.  
/ KS

Nyckelord: annihilation [14]; antimateria [16];

*

Materiens innersta-Atomer-Kärnor [1720]

Fråga:
Vad är kärnkrafter?
/Veckans fråga

Ursprunglig fråga:
Vad är kärnkrafter?
/Martin S, Dee Geer, Norrköping

Svar:
Kärnkrafter kallas också stark växelverkan. Den verkar mellan neutroner och protoner (och vissa andra kortlivade partiklar), och utgör "limmet" som håller ihop atomkärnan. Den har kort räckvidd, men är mycket stark - starkare än repulsionen mellan positiva protoner.

Ser vi det på kvarknivå, är färgkraften ansvarig för att hålla ihop de tre kvarkarna i protoner och neutroner (nukleoner). Den förmedlas här av en sorts "limpartiklar" som kallas gluoner. Teorin kallas QCD eller kvantkromodynamik (Kvantkromodynamik ). Gluonerna är masslösa (tror man) och skapas och absorberas hela tiden av kvarkarna. Se fråga 15154 för de olika typer av gluoner som finns. Detta utbyte av partiklar är likt beskrivningen av den elektromagnetiska kraften som ett utbyte av fotoner. Skillnaden är att gluonerna har färgladdning medan fotonerna är oladdade.

En annan skillnad är att avståndsberoendet är helt annorlunda. Den elektromagnetiska kraften avtar med avståndet r som 1/r2. Färgkraften är försumbar på små avstånd men mycket stor på stora avstånd (1 fm). Detta kallas asymptotisk frihet (Asymptotic_freedom ).

För atomkärnan ser man kärnkraften som en van der Waalskraft (Van_der_Waals_force ) som orsakas av att man får en polarisation av de tre färgerna i nukleonerna. Teorierna som beskriver detta är dels exakta (QCD_lattice_model ) och dels störningsteorier. Man har emellertid ännu inte kommit fram till en helt tillfredsställande beskrivning.

Alternativt kan man beskriva kärnkraften som ett utbyte av pioner. Denna klassiska modell (Yukawa_interaction ) används fortfarande och ger bra resultat. Vid utbyte av laddade pioner byter nukleonerna skepnad, dvs protoner blir neutroner och tvärtom. Se figuren i länk 1.

Utbyteskraften i atomkärnor illustreras i figuren nedan (Strong_interaction ) med följande figurtext:

An animation of the nuclear force (or residual strong force) interaction between a proton and a neutron. The small colored double circles are gluons, which can be seen binding the proton and neutron together. These gluons also hold the quark-antiquark combination called the pion together, and thus help transmit the a residual part of the strong force even between colourless hadrons.

Observera att figuren visar utbyte av en oladdad pion, vilken naturligtvis inte ger förändring proton-neutron.

Du kan läsa mer om det här: The ABC's of Nuclear Science och färgkraften .

Tack Kevin för en lektion i QCD för kärnfysiker!



/KS/lpe

Se även fråga 1433

Nyckelord: kärnkrafter [7]; färgkraften [8]; gluoner [7];

1 http://www.nobelprize.org/educational/physics/matter/14.html

*

Materiens innersta-Atomer-Kärnor [2718]

Fråga:
Jag vet att det finns kvarkar, elektroner och fotoner, och enligt det vi har lärt oss än så länge är det allt. Men what about baryonerna, mesonerna och alla de andra? Vad är de?
/daniel Z, matteusskolan, sthlm

Svar:
Baryoner och mesoner kallas med ett gemensamt namn hadroner. Det är de partiklar, som känner av den starka växelverkan (kärnkraften). De är sammansatta av kvarkar och antikvarkar.

Baryoner:

Består av tre kvarkar. Finns i laddning -- - 0 + ++. En stabil baryon: protonen.

Mesoner:

Består av en kvark och en antikvark. Finns i laddning - 0 +. Ingen stabil meson.
/KS

Se även fråga 1424

*

Materiens innersta-Atomer-Kärnor [3334]

Fråga:
vad är en atom?
/jan k, rinman, eskilstuna

Svar:
Ordet atom är egentligen grekiska. "Tomos" betyder kniv, och "a" är en negation, så atom betyder odelbar, materiens minsta beståndsdel. Redan de gamla grekerna spekulerade över atomer men det var först genom kemins framsteg på 1700 och 1800-talet som ideerna fick substans. För ungefär 100 år sedan, upptäckte Rutherford att atomen inte är en strukturlös kula. Över 99.9 % av massan är koncentrerad till en liten, positivt laddad kärna. Den negativa laddningen återfinns i ett luftigt elektronmoln runt omkring. Nu vet vi att inte heller atomkärnan är strukturlös, den består av protoner och neutroner, som i sin tur består av kvarkar. Atomen är mycket liten. I 1 g vatten finns 1000000000000000000000000 ( =1023 ) atomer.

Alltså: Atomen är inte odelbar, det är inte materiens minsta beståndsdel.  
/KS

*

Partiklar [3456]

Fråga:
Varför kan kvarkar endast existera i par och inte ensamma.
/Thomas S, Katrinelunds skola, Sundsvall

Svar:
Vi är vana vi att kraften avtar med avståndet, till exempel elektriska fält och gravitationsfält. Så är det inte för det så kallade "färgfältet", som verkar mellan kvarkar, där ökar kraften med avståndet. Tänk dig en meson som består av ett kvark-antikvarkpar. Försöker vi dra isär kvarken och antikvarken, blir det mer energi i fältet ju längre bort de kommer från varandra. Till slut är energin i fältet så hög, att det räcker till att skapa ett nytt kvark-antikvarkpar. Vi har fått två mesoner, men vi fick inte några fria kvarkar.

Kvarkar kan finnas i tripler också. Sådana partiklar kallas baryoner, som protonen och neutronen. 
/KS

*

Materiens innersta-Atomer-Kärnor [3535]

Fråga:
Vad är en kvark och vad består den av?
/Nils A, dalvik, jönköping

Svar:
Kvarkarna betraktas som riktiga elementarpartiklar, de är punktformiga och strukturlösa så vitt vi vet. Atomkärnor består ju av protoner och och neutroner, som i sin tur är uppbyggda av kvarkar.

Sök på kvark i denna databas! Där finns ganska många frågor och svar. 
/KS

*

Materiens innersta-Atomer-Kärnor [3557]

Fråga:
Hej! Jag fick en intressant fråga av en elev angående antimateria, nämligen om det är en korrekt definition eftersom han ansåg att något som hette anti borde vara raka motsatsen. Han avsåg då det faktum att antimateria har massa, precis som materia, och att antimateria därför borde kallas för något annat. Vad svarar jag på en sådan fråga?
/Andreas L, Mikaelskolan, Sollentuna

Svar:
Det är inte alla egenskaper som blir tvärtom för antimateria.  Av mycket fundamentala symmetriprinciper följer att massan hos en partikel och dess antipartikel är lika, till och med exakt lika. Något undantag är inte känt.

Vad som byter tecken är till exempel elektrisk laddning. Som följd av detta byter också det magnetiska momentet tecken. Sedan finns ett par andra kvanttal som byter tecken. Till exempel har elektronen leptontal +1, medan positronens är -1. Andra exempel är kvarkarnas "flavour" och neutrinons helicitet, men att gå in på det här i detalj skulle nog bli alltför tekniskt.
/KS

*

Materiens innersta-Atomer-Kärnor [3696]

Fråga:
Vad kallas de partiklar som är mindre än elementarpartiklarna och vad är de för nåt?
/Sandra E, Hellbergsskolan, Linköping

Svar:
Atomkärnor består ju av protoner och neutroner, eller med ett gemensamt namn, nukleoner. Förr i tiden trodde man att de var elementarpartiklar. Numera vet vi att de är sammansatta, ganska komplicerade partiklar. De består av så kallade kvarkar, som hålls ihop av "lim-partiklar", gluoner. Energikoncentrationen stör dessutom tomrummet (vakuum), som klart bidrar till dessa partiklars egenskaper. Numera anser man att kvarkar och leptoner är de egentliga elementarpartiklarna. Exempel på en lepton är elektronen. Dessutom finns förmedlarpartiklar av olika slag, t ex fotoner och gluoner. 
/KS

*

Universum-Solen-Planeterna [3716]

Fråga:
Jag undrar i vilken ordning naturkrafterna kommer i vilken är starkast och vilken är svagast?
/Caroline W, tycho brahe skolan, helsingborg

Svar:
De fyra fundamentala naturkrafterna eller bara växelverkan är de fysikaliska fenomen där partiklar påverkar varandra med krafter. Enligt nuvarande teorier kan all interaktion förklaras utifrån dessa fyra krafter.

Av de fyra fundamentala kraftverkningarna är gravitationen svagast och den starka växelverkan (kärnkrafterna) starkast. Här är ordningen (vid måttliga energier), kraftförmedlaren samt vilomassa och spinn för kraftförmedlaren:

  1. stark växelverkan (håller ihop kvarkarna i nukleoner och orsakar bindningen i atomkärnor), gluon, 0, 1
  2. elektromagnetisk växelverkan (håller ihop atomen), foton, 0, 1
  3. svag växelverkan (betasönderfall), W+ , W- och Z0 bosoner, 80-90 GeV/c2, 1
  4. gravitation (håller ihop planeter, solsystem, galaxer och galaxhopar), graviton, 0, 2

De approximativa relativa styrkorna framgår av nedanstående figur från Fundamental_växelverkan .

Se vidare fråga 957 , Fundamental_interaction och länk 1 om kraftförmedlarna.



/KS/lpe

Nyckelord: kraftverkningar [9];

1 http://hyperphysics.phy-astr.gsu.edu/hbase/particles/expar.html

*

Partiklar [3915]

Fråga:
Hejsan; 1) Hur reell och säker är existensen (i fysisk mening) av objekt mindre än atomen (t.ex. kvarkar)? 2) Finns det någon teori om monopoler?
/Hananja

Svar:
1. Sedan lång tid tillbaka har man vetat att i protonen och neutronen finns små hårda partiklar (först kallade man dem partoner). Det fick man reda på genom att skjuta in högenergetiska elektroner, och mäta hur de spred sig. Ett experiment helt analogt med Rutherfords upptäckt av atomkärnan för nästan 100 år sedan. Sedan dess har både teorier och experiment gjort stora framsteg. Man kan säga att för den som arbetar inom området, är kvarkarna ytterst reella. Tråkigt bara, att vi inte kan visa något "spår" av en kvark. De blir ju aldrig fria.

2. Magnetiska monopoler förbjuds inte av Maxwells ekvationer, men man får göra vissa förändringar, se Magnetic monopole . Det är dessa ekvationer, som är grundläggande för den klassiska elektromagnetismen. Vidare ingår magnetiska monopoler i vissa GUT-teorier (Grand Unification Theory). Ingen magnetisk monopol har påvisats.
/KS/lpe

Nyckelord: magnetisk monopol [4];

*

Partiklar [3931]

Fråga:
Har kvarkar verkligen färger?
/Veckans fråga

Ursprunglig fråga:
Hej igen! Såg i min fysikbok att Kvarkar var avbildade i färger, blå, röd, grön. Har de verkligen dessa färger eller visas de bara så för att lättare kunna skilja på dem? Jag menar är inte kvarkar så pass mycket mindre än våglängderna för synligt ljus att de inte borde ha olika färger i våra ögon?..
/Oscar L, Vadsbogymnasiet, Mariestad

Svar:
Dina synpunkter är alldeles riktiga, kvarkarna har naturligtvis inga färger. Det är en annan egenskap som man betecknar med "färg", nämligen en sorts laddning.

Det finns två sorters vanlig (elektrisk) laddning, + och -. Olika laddningar attraheras, lika stöts bort. Om man lägger ihop en enhet positiv laddning och en enhet negativ laddning så blir resultatet ingen laddning alls, och ingen elektrisk kraftverkan med omgivningen. Eftersom den elektriska kraften är ganska stark, så drar ett laddat föremål till sig den motsatta laddningen. Detta är anledningen till att de flesta föremålen omkring oss är oladdade.

Kvarkar hålls samman av något som kallas den starka kraften eller färgkraften. Denna har den udda egenskapen att ha tre laddningar. Vad skall vi kalla dessa? Jo, vi är ju vana att addera grundfärgerna RGB till andra färger och vitt ljus. Så vi kallar helt enkelt kvark-laddningarna (hm, ett problem här är att kvarkarna har vanliga laddningar också, men låt oss bortse från det) rött, grönt och blått.

Partiklar (baryoner, mesoner) måste vara ofärgade eftersom färgkraften är så stark. Det kostar helt enkelt för mycket energi för att separera färgerna.

En proton består av tre kvarkar (bilden i mitten nedan), med färgerna blå, röd och grön. Blandar man lika mängder av dessa ljussorter, får man vitt ljus. En meson består av en kvark och en antikvark (bilden till vänster), som till exempel kan ha färgerna grön och antigrön, vilket blir ofärgat. Det finns alltså antifärger i detta sammanhang. De har ingen motsvarighet i vår vardagstillvaro. Anledningen till att alla partiklar måste vara färglösa är att en färgad partikel omedelbart drar åt sig färger för att kompensera sin färg.

Färgkraften har den märkliga egenskapen att den är mycket svag vid små avstånd och blir stark först när avståndet närmar sig 1 fm (storleken av en nukleon). Detta medför två saker:

1 Kvarkarna är i stort sett fria inne i en nukleon eller meson (asymptotisk frihet, Asymptotic_freedom ). Kraften mellan kvarkarna är alltså noll för avstånd som är mindre än c:a 1 fm, medan kraften blir mycket starkt attraktiv vid större avstånd.

2 Man kan inte observera fria kvarkar eftersom färgkraften på stora avstånd motsvarar så hög energi att det bildas mesoner som består av kvark-antikvark par, se nedanstående figur där R, G, och B symboliserar kvarkar.

nukleon
(R  G  B)
.
försök att ta loss den blå kvarken
R  G    (B  | antiB)  B
kvark/antikvark inom parantesen skapas
.
slutresultatet
(R  G  B)     (antiB  B)
 nukleon       meson     (båda färglösa)

Eftersom kraftförmedlarpartiklarna, gluonerna, har en färg och en annan anti-färg (se fråga 15154 nedan), innebär utbytet av gluoner att kvarkarna hela tiden skiftar färg.

Se vidare Strong_interaction , Quantum_chromodynamics , Quark och Gluon . Bilden är från Wikimedia Commons.



/KS/lpe

Nyckelord: standardmodellen [24]; färgkraften [8]; kvark [12]; gluoner [7];

*

Partiklar [4220]

Fråga:
Jag läste någonstans att forskare som hade byggt en ny partikelaccelerator för att bekräfta teorin om Big Bang skulle börja testa vilka urpartiklar som bildas redan för ett par månader sedan. Jag undrar om ni känner till något resultat om det.
/Marcus A

Svar:
Protonen består av tre kvarkar, men ingen har lyckats slå ut någon av dem. Det finns teoretiska förklaringar varför det inte går. Samma teorier gör en annan intressant förutsägelse. Klämmer vi ihop eller hettar vi upp en atomkärna tillräckligt, befrias kvarkarna i en viss mening, de är inte bundna till enskilda partiklar längre. Vi har fått ett nytt fastillstånd hos kärnmaterien, ett så kallat kvark-gluon plasma. Enligt Big Bang teorin var Universum i detta tillstånd i ett tidigt stadium.

Det man är ute efter, är att producera kvark-gluon plasma genom att låta tunga atomkärnor kollidera vid mycket höga energier. Flera sådana experiment är igång vid BNL, Brookhaven, Long Island, New York. Där kolliderar guldkärnor som accelererats till 99.95 % av ljushastigheten. Acceleratorn är ringformad och är 1 km i diameter. Det kan bildas femtusen partiklar i en sådan kollision. Det är genom att studera dessa partiklar, man hoppas kunna avgöra om ett kvark-gluon plasma bildats. Plasmat går inte att se direkt. Om det bildas, kan man räkna med att det sönderfaller till partiklar efter ungefär 0.000000000000000000001 s.

Ungefär år 2005 räknar man vara klar med en ännu större anläggning vid CERN, Geneve. Där kommer man kollidera blykärnor vid ytterligare 20 gånger högre energi.

Dessa experiment är inte något test av Big Bang-modellen, de är snarare test av teorierna för kvarkar och gluoner.
/KS

Nyckelord: kvark [12];

*

Partiklar [4293]

Fråga:
Har protoner och gluoner alltid samma vikt?? och Kvarkar är ju världens minsta partikel - men - Borde inte gluoner vara mindre än kvarkar?
/Andreas D, Kristinedal, Stenungsund

Svar:
Eftersom protonen består av kvarkar och gluoner måste protonen vara tyngre än gluonen. Enligt vad man tror nu saknar gluonen vilomassa. Både gluoner och kvarkar betraktas som strukturlösa (=punktformiga).
/KS

*

Partiklar [4812]

Fråga:
Jag undrar när allt tar slut. Atomen är inte det minsta som finns, det finns ju kvarkar mm och nu har man hittat vad kvarkar är uppbyggda av, när ska dom komma till den minsta delen? Eller är det en enda stor oändlighet?
/Jakob Ö, Taserud, Arvika

Svar:
Såvitt vi vet har man ännu inte hittat någon inre struktur hos kvarkarna. Sök på punktformig i denna databas, så får du frågan belyst.
/KS

*

Partiklar [5683]

Fråga:
jag undrar varför neutronerna inte har någon laddning?
/nils r, kungshögsskolan, ljungby

Svar:
Neutronen består i huvudsak av en uppkvark (laddning +2/3) och två nedkvarkarkar (laddning -1/3). Summan blir alltså 0.
/KS

*

Värme [6400]

Fråga:
Kan det bli hur varmt som helst? Hur snabbt hoppar molekyler och atomer omkring när det är som varmast? Kan de röra sig fortare än ljusets hastighet?
/fredrik h, helsingborg

Svar:
När det är som varmast finns inte molekyler, atomer eller atomkärnor. Ja till och med protoner och neutroner går sönder. Bara kvarkar och gluoner och några andra fundamentala partiklar existerar. Tillståndet kallas kvark-gluon plasma.
/KS

Se även fråga 5956

*

Partiklar [7298]

Fråga:
I våran fysikbok står att man inte kan dela en elektron?? finns det en laddning som är mindre än en elktron??
/Sanaa A, Karlbergskolan, Åmål

Svar:
Enligt vår nuvarande kunskap är elektronen punktformig och odelbar, alltså en äkta elementarpartikel. Kvarkar (som protoner och neutroner består av) har 1/3 eller 2/3 elektronladdningar, men de har aldrig observerats fria.
/KS

*

Partiklar [7437]

Fråga:
vilken är den minsta partikel som man känner till?
/hannes k, näsbydalskolan, Täby

Svar:
I standardmodellen betraktas kvarkar och leptoner som punktformiga och strukturlösa, alltså riktiga elementarpartiklar. Experimentellt är det kollat ner till 10-18 m, eller en tusendels protondiameter. Sök efter dessa ord i denna databas!

Med minst skulle man också kunna mena lättast. Då är det säkert elektronneutrinen (en neutral lepton). Massan skulle kunna vara noll, men vissa mätningar kan tolkas som att den inte är det. I varje fall är den mycket lätt, mindre än en miljontedels elektronmassa.
/KS

*

Elektricitet-Magnetism [7457]

Fråga:
hej Jag undrar vad en laddning egentligen är för något och hur den uppstår? Jag vet att kvarkar är plus- och minusladdade men hur blir de det?
/Niklas E, Näsbydalskolan, Täby

Svar:
Det här går nog inte att svara på. Som fysiker får man nog nöja sig med att konstatera att världen uppför sig på det viset, och försöka beskriva det så bra som möjligt. Fanns inga elektriska laddningar, skulle världen se helt annorlunda ut.
/KS

*

Partiklar [7548]

Fråga:

1) På vilket sätt ger Higgspartikeln uphov till det som vi uppfattar som massa?

2) Om man inte hittar partikeln, så säger man att hela Standardmodellen rasar, vad menar man med det?

3) Hur tror ni att man kommer se Higgs i den nya LHC:n (Large Hydrone Colider), som man håller på att bygga på CERN i Schweiz?

4) Vem uppfann Higgs??Jag antar att det var en man som hette Higgs??..eller?
/Danne F, Katedralskolan, Uppsala

Svar:

1. Sajt 1 försöker förklara det på enkel svenska. Sajt 2 är på engelska.

2. Då måste vi hitta något annan mekanism som genererar partikelmassor.

3. Higgspartikeln väntas mest sönderfalla till 2 b-kvarkar, som vardera vanligen sönderfaller till två strålar av hadroner (starkt växelverkande prtiklar). Man kommer leta efter kollisioner där det finns 4 hadronstrålar.

4. Peter Higgs är engelsman. Han var inte alldeles ensam om detta, så är det för det mesta.
/KS/lpe

Se även fråga 1375

Nyckelord: higgspartikeln [10]; standardmodellen [24];

1 http://courses.physics.kth.se/5A1310/elementar/symmetrier.html
2 http://atlas.web.cern.ch/Atlas/documentation/EDUC/physics9.html

*

Partiklar [7597]

Fråga:
Finns gluonerna i kvarkarna eller rör de sig fritt inom en partikel?
/Dzevdan K, Gullstrandskolan, Landskrona

Svar:
Gluoner är förmedlarpartiklar som förmedlar krafterna mellan kvarkarna.
/KS

*

Partiklar [8517]

Fråga:
Hej igen. Detta kanske börjar bli lite tjatigt, men faktan om anti materia på internet är minst sagt tvetydig. På vissa sidor står det att det är bara laddning, andra laddning och spin och andra att det är samtliga kvanttal och andra att det är laddning och baryon/lepton tal. på britannica.com står det t.ex: "The electrical properties of antimatter are opposite to those of ordinary matter; thus, for example, the antiproton (p) has a negative charge, and the antineutron (n), although electrically neutral, has a magnetic moment opposite in sign to that of the neutron."

Magnetiskt momentum är ju spin. Så skulle ni kunna berätta exakt vilka egenskaper som skiljer materia från anti materia?
/Pelle U

Svar:
Magnetiskt moment och spinn är relaterade, men det är inte samma sak. Man skulle kunna tro att protonens spinn på något sätt genererades av kvarkarnas spinn. Experiment visar att bara c:a 25% av protonens spinn har med kvarkarna att göra. Resten genereras på något annat sätt, vi vet inte hur. Det planeras experiment vid RHIC (Long Island) med polariserade protonstrålar. Där hoppas man kunna kasta ljus över hur protonens spinn uppkommer.

Som exempel på att dessa saker inte är alldeles enkla, kan vi ta den neutrala K-mesonen. De neutrala K-mesonerna uppträder aldrig som fria partiklar och antipartiklar. I stället förekommer de som linjärkombinationer av båda tillstånden. Två linjärkombinationer är möjliga, symmetrisk och antisymmetrisk. De kallas KL och KS.
L står för Long och S står för Short. Det är halveringstiden man syftar på. Dessa saker går inte att begripa på något intuitivt sätt. Det måste beskrivas kvantmekaniskt.
/KS

*

Partiklar [8619]

Fråga:
1.Vad är partiklar? 2.Vad består partiklar av?
/Henke n, kungshöga, Mjölby

Svar:
I teorierna för elektrosvag och stark växelverkan betraktas elementarpartiklarna (kvarkar och leptoner) som punktformiga och strukturlösa. Numera arbetar man men andra teorier, där partiklarna uppfattes som vibrerande strängar i ett rum med 10 eller 11 dimensioner. Kolla svaren nedan!

Vill du veta mer om detta, försök få tag på boken Ett utsökt universum av Brian Greene (översättning Hans-Uno Bengtsson).
/KS

Se även fråga 8357 och fråga 2688

*

Materiens innersta-Atomer-Kärnor [8750]

Fråga:
Hejsan! Jag har fått veta att när vätet i solen slås ihop till helium omvandlas en del av dess materia till engergi och att när heliumet är slut fortsätter det med att heliumet slås ihop till ännu tyngre ämen till det är framme vid järn. Vad jag inte förstår är hur det kan komma sig. Det ända sättet jag kan få det att stämma är ifall en proton eller neutron inte alltid har lika stor massa. Men hur kan man i så fall säga sig veta massan för dessa partiklar ifall de varierar? Och eftersom de är uppbyggda av kvarkar måste ju det i så fall innebära att alla kvarkar inte heller har lika stor massa. Stämmer det? Är det verklige så? Eller är det så men skillnaden är så liten att den är försumbar? Och påverkar det i så fall kärnans elektriska laddning?

Tacksam för svar!
/Torbjörn 0, Malmö

Svar:
Massan av två protoner plus två neutroner är större än massan av en heliumkärna. Skillnaden består av bindningsenergi, och härifrån tar solen den energi den strålar ut.

Vad beträffar dessa partiklars massa är det en invecklad historia, som bara delvis är förstådd. Kvarkarna i en proton utgör en liten del av protonens massa.
/KS

*

Partiklar [9049]

Fråga:
Jeg undret om dere ikke kunne forklare meg hvordan Paulis utelukkelsesprinsipp fungerer... Det sies at fermioner ikke kan være i samme tilstand på samme tid? Og ut i fra det kan man forklare at elektronene i atomer legger seg i flere lag. Og, hvorfor kan de ikke være i samme tilstand til samme tid?
/Vanja M, Horten vederegående skole, Horten

Svar:
Svaret på den sista frågan är enkelt: Vi vet inte varför. Alla fundamentala materiepartiklar (kvarkar och leptoner) har halvtaligt spin och är därmed fermioner. En framgångsrik teori måste inkludera detta faktum, men kan därför inte sägas ha förklarat varför det är så.

Sammansatta partiklar kan ha heltaligt spinn (bosoner). 4He och 3He har mycket olika egenskaper vid låga temperaturer. Det beror på att de har spinn 0 och 1/2 respektive.
/KS

*

Partiklar [9255]

Fråga:
Hvis du på en eller annen merkverdig måte skulle kunne klare å dele et elektron, ville de to delen da fått en halv elementærladning hver, eller en hel?

Oppkvarken har visstnok 9,8 elektronmasser, og nedkvarken 15,7. Hvordan har det seg da at de alltid omtales som punkter, som om de ikke hadde volum? Et proton skal bestå av to oppkvarker og en nedkvark. Hvis du da legger sammen disse massene, gjør de ikke opp for protonmassen. Hvorfor ikke? Protonet har ikke noe masse i seg selv? Som en vegg rundt kvarkene? Og gluonene veier vel ingen ting? Takker for svar
/Vanja M, Horten vgs, Horten

Svar:
Såvitt vi vet går inte elektronen att dela. Den uppfattas som en äkta elementarpartikel.

De lätta kvarkarnas massor ska du inte ta allvarligt på. Det finns inga metoder att direkt mäta dem, och olika sätt att beräkna dem ger olika resultat. Protonens valenskvarkar är som du skriver, men protonens inre struktur är mycket mera komplicerad än så. Vi skickar en bild från CERN Courier, som visar vad man måste ta hänsyn till vid en approximativ beräkning av protonens spin. Visste du att 5% av dig är antimateria?
/KS

1 http://www.cerncourier.com/main/article/41/9/18/2/cerndes2_9-01

*

Partiklar [9266]

Fråga:
Det ble skrevet at 5% av mennesket er antimaterie. Men er det da ikke slik at når materie og antimaterie møtes, så skjer der en slags eksplosjon og massevis av energi fyker rundt, dvs partikkelen og antipartikkelen blir til ren energi? Takknemmelig for svar.
/vanja m, horten vgs, Horten

Svar:
Hela tiden hoppar det upp kvark-antikvark par som sedan snabbt annihileras (förintas). Energin som då frigörs, används för att "betala" den energi som behövdes för att bilda partiklarna. Dessa kvarkar kallas sjökvarkar (engelska: sea quarks), och utgör ungefär 10% av kvarkinnehållet i en proton. Alltså, skjuter vi in elektroner i en proton, finner vi att i var tionde elektron-kvark kollision är en sjökvark inblandad.
/KS

Se även fråga 9255

Nyckelord: annihilation [14]; parbildning [7];

*

Partiklar [9747]

Fråga:
Har studerat tidigare svar om spinn i frågelådan och har följande frågor.

1- Om man inte vet vad spinn är, hur kan man då påstå att en viss partikel har t.ex. ½-taligt spinn eller heltaligt?

2- Kvarkarna som inte är fria och därmed omätbara har fått spinnkvanttalet ½, varför då?

3- Mesonerna har fått 0-spinn, varför det?
/C-Anders K, Sollefteå

Svar:
Att elektronen har rörelsemängdsmoment blev man klar över under 1920-talet. Elektronens spinn mättes direkt i Stern-Gerlach experimentet 1922 till ½. Spinn ska inte uppfattas som en rotation, det är ett rent kvantmekaniskt fenomen, som inte har någon klassisk motsvarighet. Spinnet och dess egenskaper faller naturligt ut ur den relativistiska kvantfätsteori som utarbetades av Paul Dirac. Att gå in på det ligger långt utanför denna frågelådas ambitioner. Likaså hur man man bestämmer spinn för olika partiklar.

Det finns fortfarande många olösta frågor om partiklars spinn. Protonens spin är ½. Det tycks vara lätt att förklara, protonen består ju av 3 kvarkar, vardera med spinn ½. Vi får då ekvationen ½ -½ + ½ = ½. Så enkelt är det tyvärr inte. Experiment har nyligen visat, att kvarkarnas bidrag till protonens spinn bara är 20 - 30%. Varifrån kommer resten? Det här är inga enkla saker.
/KS

1 http://hyperphysics.phy-astr.gsu.edu/hbase/spin.html

*

Partiklar [10116]

Fråga:
En fråga om partiklar den här gången:

Atomen är ju mestadels tomrum, den volym som upptas av kärnan är ju nästan försumbar. Hur är förhållandet för själva nukleonerna - dvs hur mycket är tomrum mellan de kvarkar som bygger upp protoner, neutroner, osv?

Har man någon teori om vad kvarkarna är uppbyggda av eller anses dessa vara de minsta partiklar som (hittills) upptäckts?
/Bengt I, Sven Eriksson, Borås

Svar:
Om atomen mest består av tomrum kan verkligen diskuteras. I Bohrs atommodell är det så, men den modellen är övergiven sedan mer än 3 generationer. I den kvantmekaniska beskrivningen av den neutrala väteatomen finns elektronen överallt. Sedan är det en annan sak att atomkärnan upptar en mycket liten del av atomens volym.

I standardmodellen för elementarpartiklar betraktas leptoner (som elektronen) och kvarkar som strukturlösa (punktformiga). Det betyder inte att de är lokaliserade i en punkt i rummet, det förbjuder obestämdhetsrelationen. Det betyder att kvarkarna och gluonerna i en proton befinner sig överallt. Frågan om tomrummet i protonen är alltså meningslös ur kvantmekanisk synpunkt.

Man arbetar med att förena allmänna relativitetsteorin med kvantmekaniken. Dessa teorier kallas supersträngteorier. Här är elementarpartiklarna små vibrerande strängar, alltså inte längre punktformiga. Detta är bara fråga om teorier, alltså inte några experimentella resultat.
/KS

Se även fråga 3652 och fråga 9467

*

Partiklar [10288]

Fråga:
Vad kommer egentligen nukleoners massa ifrån. Hur kan dessa partiklar ha massa. Kommer detta från energin de innehar? väger en elektron 0,0054858 u utan materia eller är det dess innebärande energitillstånd som gör att den väger?
/Nicole d, Lunds gymnasium, Lund

Svar:
En proton innehåller kvarkar, antikvarkar och gluoner. Du förmodar alldeles rätt. Protonens massa består i huvudsak av dessa partiklars energi.

Elektronens massa har man för närvarade ingen förklaring på. Den så kallade "standardmodellen" säger inget om det. Det är en anledning till att man letar efter en djupare, bakomliggande teori.
/KS

*

Elektricitet-Magnetism [10405]

Fråga:
vad är protoner uppbyggda av?
/tobias w, tensta, spånga

Svar:
Man brukar säga att en proton består av två uppkvarkar och en nerkvark. I själva verket är protonen mycket mera komplicerad. Där finns gluoner som förmedlar de starka krafterna. Där finns kvarkar och antikvarkar ur den så kallade sjön. Kolla bilden!
/KS

1 http://www.cerncourier.com/main/article/41/9/18/2/cerndes2_9-01

*

Kraft-Rörelse [10444]

Fråga:
Vad menas med att genom att låta tex fotoner vakuumpolariseras och växelverka med ett Higgs-fält så känner de ett mostånd och därmed är de massiva-de har massa. Vad har Higgs-partikel med motståndet i vakuum att göra? Är Higgs-partiklarna det som utgör motståndet? Är det faktum att de kan känna motstånd bevis på att de har massa?' En sista fråga: Vad menas med att gravitationen är kvantiserad och varför sägs den vara det?
/Maria E, Åsö, Stockholm

Svar:
Begreppet vakuumpolarisation hör i första hand hemma i kvantelektrodyamiken, och där finns inga Higgsfält. Alltså, det motstånd ljuset känner i vakuum genom vakuumpolarisationen (som bestämmer ljushastigheten) beror inte på något Higgsfält. I varje fall inte på något uppenbart vis.

Higgsmekanismen konstruerades för att förklara varför fotonens tyngre kompisar
( Z0,W+ och W- ) är så tunga. Också de andra elementarpartiklarna
(kvarkar och leptoner) får sina massor på detta vis. Fotonen är masslös just därför att den inte känner av Higgsfälten. Detta enligt den så kallade standardmodellen. Higgsmekanismen är ännu inte experimentellt bekräftad. Kolla sajten (på engelska).

Den väl etablerade gravitationsteorin (allmänna relativitetsteorin) går inte ihop med kvantmekaniken, och det är ett bekymmer. Man jobbar intensivt med att hitta en kvantiserad gravitationsteori, men man är inte framme. Man anar vagt något, som går under beteckningen M-teori.
/KS

Se även fråga 8357 och fråga 9467

1 http://www.sciam.com/askexpert_question.cfm?articleID=000290D2-391C-1C71-84A9809EC588EF21&catID=3&topicID=13

*

Partiklar [10728]

Fråga:
jag får tacka för svaret som jag fick på min fråga, men nu väcktes ytterligare en fråga som ni kanske kan svara på , vi skrev följande "Med en elektron och en neutron händer ingenting (vid låga energier)när dessa krockar " men vad händer om man har hög energi ?? måste bara veta hur sönderfallet blir...

tack i förskott !!

//Singh
/Manjinder S, polhemsgymnasiet, Göteborg

Svar:
Vid riktigt höga energier kolliderar elektronen med en av kvarkarna i neutronen och en massa olika saker kan hända. Vanligtvis bildas ett stort antal nya partiklar. Det var genom den sortens experiment man först påvisade att protonen och neutronen består av kvarkar.
/KS

Se även fråga 10689

*

Materiens innersta-Atomer-Kärnor [10860]

Fråga:
Hej! Jag undrar hur pioner, trots sin korta livslängd, kan förmedla den starka kraften mellan nukleoner?
/Jimmy K, Martin Koch-gymnasiet, Hedemora

Svar:
Det var den japanske fysikern Hideki Yukawa som 1935 föreslog att den starka kraften mellan nukleonerna i kärnan förmedlades av en medeltung partikel, kallad Yukawa-meson. Den blev senare identifierad som pionen. Detta är alltså en modell för lågenergetiska processer i kärnan. Det finns flera.

Vad vi inte håller med om är att pionen är kortlivad, tvärtom, den är extremt långlivad i detta sammanhang. Den karakteristiska tidsskalan bestäms av den tid det tar ljuset att passera en nukleon, och det är ungefär 10-23 s. En pion lever 1015 gånger längre, eller 10-8 s. När en pion sönderfaller, måste kvarkar ändras, och det kan inte stark växelverkan göra. Den måste sönderfalla med svag växelverkan, vilket förklarar varför den är så långlivad.

Förövrigt ska man nog inte uppfatta pionen som påtaglig i detta sammanhang. Pionen uppträder som virtuell partikel, och sådana brukar inte bry sig så mycket om lagar och regler. Detta har de Heisenbergs obestämdhetsrelation att tacka för.
/KS

*

Värme [10887]

Fråga:
Hej! Jag och några kollegor diskuterade idag vad som egentligen händer med en atom då den uppvärms. Det är ju ett välbekant faktum att en atom rör sig mer då den uppvärms och tar då större plats. Vad är det i atomen som rör sig mer? Elektronerna? Kvarkarna? Har det att göra med bindningarna mellan atomer i en molekyl? Mkt tacksam för svar! mvh Mikael
/Mikael L, Sannerudsskolan, Kil

Svar:
Man kan inte tala om temperatur för en atom, man måste ha många. I en gas kolliderar molekylerna ideligen med med varandra och utbyter energi. Temperaturen är proportionell mot gasmolekylernas medelenergi. Man kan beskriva en vätska på ett likartat sätt. I ett fast ämne vibrerar atomerna kring ett jämviktsläge. Ju högre temperatur desto kraftigare vibrationer. Till slut lösgör sig atomerna. Ämnet smälter.
/KS

*

Universum-Solen-Planeterna [11096]

Fråga:
Jo jag läste nyligen i illustrerad vetenskap om att man tror sig ha upptäckt en ny form av himlakropp nämligen en "kvarkstjärna"... alltså en himlakropp bestående av fria kvarkar. Är de inte så att om man försöker separera ett kvarkpar så krävs så mycket energi att det bildas ett nytt kvarkpar, därför existerar inte fria kvarkar?
/Erik N

Svar:
Först ska nämnas att det inte råder full enighet om tolkningen av data. Genom mätningar med röntgenteleskopet Chandra och Hubbleteleskopet, har man hittat ett par stjärnor som tycks vara mindre än vad som är möjligt för en neutronstjärna. Det skulle kunna vara kvarkstjärnor.

Antag att vi har en tät dubbelstjärna där ena stjärnan är en "vanlig" stjärna och den andra en neutronstjärna. När den första stjärnan går in i jättestjärnstadiet, kan materia överföras till neutronstjärnan. Då ökar det inre trycket och energitätheten. Vid en viss nivå "smälter" neutronerna och kvarkarna frigörs. I den processen tillförs energi utan att temperaturen ökar. Det är precis definitionen av fasövergång. Det påminner mycket om när is smälter. Om processen fortsätter, slutar det hela med ett svart hål.

I kvarkstjärnan är kvarkarna fria i den meningen att de inte är bundna till någon nukleon, men de kan inte lämna stjärnan. Den nya fasen av kärnmateria kallas kvark-gluonplasma, och studeras experimentellt vid acceleratorer här på jorden. Ännu finns inga definitiva resultat.

Sajterna nedan är på engelska. Vi har inte hittat något på svenska.
/KS

Se även fråga 4220 och fråga 5956

Nyckelord: Illustrerad Vetenskap [17];

1 http://antwrp.gsfc.nasa.gov/apod/ap020414.html
2 http://www.spaceflightnow.com/news/n0204/11newmatter/

*

Partiklar [11101]

Fråga:
Kan materia förekomma i hur små portioner som helst? En elektron har jag aldrig sett kunna delas, men kvarkar, kan de delas? Neutriner har ev massa, om än väldigt liten; kan många neutriner slås samman till en större partikel? (laddningsproblem skulle det ju inte bli!) Kan fotoner ha massa, om än liten? Kan det förekomma kraftverkan mellan fotoner? neutriner? Är även neutriner partikel-våg -dualistiska till sin karaktär?
/Thomas Å, Märstagymnasiet, Märsta

Svar:
I standardmodellen är elementatpartiklarna (kvarkar och leptoner) punktformiga och odelbara. Att neutriner skulle kunna slå sig ihop till större aggregat finns ingen anledning att tro. Den experimentella övre gränsen för fotonens massa är 0.0000000000000006 eV/c2. Jämför med elektronens massa, som är 511000 eV. Växelverkan kan förekomma mellan fotoner, men inte genom direkta processer. Växelverkan mellan neutriner är i princip möjligt, men det har bara varit av betydelse när universum var mycket ungt med mycket hög densitet. All materia har partikel- vågdualitet.
/KS

*

Partiklar [11399]

Fråga:
När antimateria och materia möts annihileras de och strålning bildas. Varför förintas då inte kvark-antikvark par? Jag läste någonstans att vi består till ca 5% av antimateria, är detta sant? Sen har jag en fråga om imaginär tid: Imaginär tid är precis som vanlig tid fast när man räknar på det används imaginära tal, detta skulle då innebära att skillnaden mellan tid och rum försvinner. Vad menas med att skillnaden mellan tid och rum försvinner??
/Erik N, Umeå

Svar:
Visst annihileras kvarkar och antikvarkar. En pimeson består av sådana partiklar, och den sönderfaller genom kvarkannihilation.

Det är riktigt att ungefär 5% av vanlig materia består av antimateria. Ungefär 10% består av kvark-antikvarkpar som hela tiden uppstår och annihileras. De kallas sjökvarkar.

Se även fråga 9266

*

Materiens innersta-Atomer-Kärnor [11529]

Fråga:
Hej! Jag skriver ett projektarbete om antimateria. Jag har stött på några frågor.

Har jag förstått det här rätt?? tex om en proton består av kvarkar. Då består en antiproton av antikvarkar? och en antikvark en kvark som har omvända kvanttal (baryontal, leptontal, färg, isospinn, särhet, hyperladdning, elektriskladdning)

Var kan man läsa om projektet med att tillverka antiväte? Jag har redan boken "kosmos 1997".

Mvh /Michael Lindahl
/Michael L

Svar:
Säger man att en proton består av tre kvarkar är det en mycket förenklad bild. 5% av protonen är faktiskt antikvarkar. Leptontalet är irrelevant för kvarkar, de är ju inte leptoner. Sök gärna på antimateria i denna databas.
/KS

Se även fråga 9255 och fråga 9747

1 http://livefromcern.web.cern.ch/livefromcern/antimatter/factory/AM-factory00.html

*

Partiklar [11817]

Fråga:
Jag har försökt att skriva på på en formel angående varför en neutron blir en proton och en elektron som sedan skjuts ut ur kärnan som en beta-partikel. Anledningen till att jag misstänker att den kräver justeringar och kanske inte äns stämmer är att vår lärare inte har tillräcklig kunskap inom ämnet för att kunna svara och brist på fakta. Tack vare en klasskamrat som letat efter svaret på internet fick jag följande fakta:

En neutron och protoner består av tre kvarkar av två olika typer, up- och down-kvarkar. Neutronen består av en up-kvark och två down-kvarkar. En proton består av två up-kvarkar och en down-kvark. En up-kvark har laddningen 2/3 medan down-kvarken har laddningen -1/3. I neutronen gäller följande:

(2/3) + (-1/3) + (-1/3) = 0

Alltså är neutronen neutral. I protonen gäller följande:

(2/3) + (2/3) + (-1/3) = 1

Alltså är protonen envärd posetiv.

Hur själva övergången från neutron till proton och elektron gick till framgick inte av den fakta som min kamrat lyckats hitta. Därför försökte jag själv att lösa det. Av faktan listad ovan kunde vi se att det som skillde neutronerna från protonerna är att en av neutronernas down-kvarkar ska bli en up-kvark. Sedan tidigare visste jag att en elektron är envärd negativ. Alltså -1. Eller -3/3. Ifrån detta kunde jag dra slutsatsen att:

(2/3) + (-3/3) = (-1/3) up + elektron = down

och

(-1/3) = (3/3) + (2/3) down = elektron + up

Där av har vi gjort en down-kvark till en up-kvark (som gjorde neutronen till en proton) och en elektron (som skjuts ut som en beta-partikel.)

Om man utgår ifrån den fakta som jag innehade så stämmer det. Det jag undrar är om det fungerar i verkligheten också.

Hoppas att jag har uttryckt mig så att ni kan förstå. Tack på förhand!
/Daniel B, Tråsätra skolan, Åkersberga

Svar:
Du har kommit till rätt slutsats, en nerkvark ska förvandlas till en uppkvark. Man känner bara en process för detta, svag växelverkan. Den förmedlas av en mycket tung förmedlarpartikel, W-bosonen. Den är faktiskt 80 gånger tyngre än protonen. Ytterligare en partikel är inblandad. Vid sönderfallet bildas en antineutrino. Detta för att antalet partiklar av elektrontyp (leptoner) ska bevaras.
/KS

Se även fråga 11001

*

Materiens innersta-Atomer-Kärnor [14183]

Fråga:
Noterar att det i en del sammanhang dyker upp uttrycket 'kollapsar vågfunktionen'. Vad innebär att en funktion kollapsar? Division med noll? Vilka funktioner kan överhuvudtaget 'kollapsa'? Och vad är det som är så speciellt med det? Är följderna av en 'kollaps' positiva eller negativa?
/Thomas Å, Märstagymnasiet, Märsta

Svar:
Thomas!

Det gäller mycket inom kvantmekaniken att när det kommer till en exakt förståelse eller tolkning av de matematiska uttrycken så återstår mycket och diskussionerna som påbörjades av Bohr och Einstein pågår ännu. Någon lär ha sagt att om du tror du förstått kvantmekaniken så har du inte förstått någonting!

Vad som däremot är klart är att kvantmekaniken fungerar alldeles utmärkt när vi räknar på fysikaliska system. Det som kvantmekaniken kan förutsäga (sannolikheter att ett system befinner sig i olika tillstånd och sannolikheter att ett system övergår från ett tillstånd till ett annat) förutsägs mycket väl.

I fysikaliska sammanhang "lånar" man ofta ord från vardagsvärden. Det finns oftast en likhet mellan begreppen, men man får inte övertolka och tro att ordet betyder exakt samma i den fysikaliska värden.

Ett exempel är att kvarkar har "färg". Detta får inte tolkas bokstavligt så att kvarkar är färgade utan så att kvarkarna har en sorts "laddning" som har egenskapen att tre olika laddningar (färger) blir tillsammans laddningsfria på samma sätt som tre grundfärger rött, grönt och blått tillsammans blir vitt.

För att äntligen komma till kollaps av en vågfunktion som du frågade om: varje kvantmekaiskt system kan beskrivas av en vågfunktion. Denna talar om sannolikheten för att systemet ser ut si och sannolikheten för att det ser ut så:

psi = A*psi(si) + B*psi(så) (1)

A och B kallas för amplituder och sannolikheten att systemet ser ut på ett visst sätt ges av |A|2 respektive |B|2 (absolutbeloppen behövs därför att amplituderna kan vara komplexa). För detta fallet gäller då förstås eftersom alla sannolikheter skall summera sig til 1:

|A|2 + |B|2 = 1 (2)

Om vi nu observerar systemet och t.ex. finner att det befinner sig i tillståndet si, så beskrivs systemet inte av vågfunktionen (1) utan av

psi = 1*psi(si) (3)

Amplituden för si har alltså blivit 1 och amplituden för så har blivit 0. Det är detta fenomen att en observation (mätning) får vågfunktionen att gå från (1) till (3) som kallas att vågfunktionen kollapsar.
/Peter E

Nyckelord: vågfunktion [2]; kvantmekanik [30];

*

Blandat [15996]

Fråga:
Hej! Till vardags säger vi oftast att solskenet har sin grund i att solen förbränner/fusionerar väte till helium. Det är ju också rejält hett i solens inre. Solen lär ju därför inte bestå av neutrala väteatomer. Men har man koll på solens laddningsfördelning? Är den neutral eler är den då och då negativt laddad efter att ha slungat ut partiklar av t ex enbart positiv karaktär? Är det protoner i solens inre eller kan det där finnas fria kvarkar o andra partiklar som inte finns fria i vår kalla värld? Går dessa att studera via ljuset - eller är redan solytan "för kall"?
/Thomas Å, Arlandagymnasiet, Märsta

Svar:
Thomas! Laddningsvariationen måste vara noll eftersom plus och minusladdningar skulle attrahera varandra starkt. Temperaturen i solens inre är c:a 15 miljoner K, och då är alla atomer helt joniserade. Atomkärnor och motsvarande antal elektroner rör sig alltså fritt. Det vi ser från solen är ytan eller fotosfären med en temperatur på 5800 K. Där är atomerna bara delvis joniserade.

Solens nettoladdning är nära noll - både protoner och elektroner slungas ut. Om solen fick för hög laddning skulle den eftersom den elekriska kraften är stark och har oändlig räckvidd dra till sig laddningar så den blev neutral.

Nej, energin i solens inre är inte tillräcklig för att skapa fria kvarkar.
/Peter E

*

Partiklar [11969]

Fråga:
Skulle ni kunna utöka min kunskap angående partiklar genom att i svaret på denna fråga skriva upp alla idag kända partiklar och ge en ingående beskrivning av dessa.
/Daniel B, Tråsätra skolan, Åkersberga

Svar:
Att räkna upp alla idag kända partiklarna och deras egenskaper låter sig inte göras här. Det är också alldeles onödigt, för det finns redan på nätet. Informationen där uppdateras ständigt av Particle Data Group , så den är alltid aktuell. Vi kan i alla fall ge vilka olika typer av partiklar man räknar med.

A: Förmedlarpariklar:

Gravitonen förmedlar gravitation (ej påvisad).

Fotonen, W+, W-, Z0 förmedlar elektromagnetisk och svag växelverkan.

8 st gluoner förmedlar stark växelverkan.

B: Elementarpartiklar:

Leptoner, 6 st och deras antipartiklar (exempel: elektronen).

Kvarkar, 6 st och deras antipartiklar.

C: Två olika sorts hadroner kan bildas av kvarkar:

Mesoner som består av en kvark och en antikvark. Man känner ungefär 100 sådana.

Baryoner som består av 3 kvarkar. Man känner ungefär 30 sådana och deras antipartiklar.

D: Hätill kan läggas Higgs-bosonen som har en alldeles speciell roll. Den gör att elementarpartiklar har massa. Den är ännu inte påvisad.
/KS

Se även fråga 11946

*

Materiens innersta-Atomer-Kärnor [12070]

Fråga:
Hur kan man veta att saker existerar som man inte kan se? Ta tex atomens uppbyggnad med dess elektroner eller kvarkar osv, dvs sånt som är för smått för dagens teknik att se.
/Jens G, Läringeskolan, Åtvidaberg

Svar:
Att "se" betyder bara att man uppfattar något med det mänskliga ögat. Man ser ett föremål antingen genom att det strålar ut ljus eller att det reflekterar ljus. Hur skarpt man kan se beror dels på detektorns kvalité (ögat) och dels på ljusets våglängd. Ögats upplösning kan man förbättra med optiska anordningar (t.ex. mikroskop), men begränsningen som sätts av ljusets våglängd går inte att komma runt - föremål som är mycket mindre än ljusets våglängd kan man inte se.

Hur skall man komma runt detta och kunna mäta även på mindre föremål? Jo, genom att använda "ljus" av kortare våglängd. Man kan även använda sig av partiklar, eftersom dessa även har vågegenskaper (elektronmikroskop). Vad man behöver är dels en källa för kortvågigare strålning (t.ex. accelerator, röntgenrör) och dels en detektor för strålningen - ögat är ju bara känsligt för synligt ljus. Med hjälp av elektroner med mycket hög energi - kort våglängd - har man indirekt men tydligt "sett" att protonen innehåller tre masspunkter - kvarkar.
/lpe

Nyckelord: accelerator [7];

*

Kraft-Rörelse, Partiklar, Universum-Solen-Planeterna [12080]

Fråga:
Vad är egentligen den "starka kraften" för något? Kan man jämföra den kraften med gravitationen?

Och måste planetbanor var eliptiska? Finns det ingen möjlighet att en planet kan ha en absolut rund bana runt någon stjärna?
/Mats D, Göteborg

Svar:
Den starka kraften är den kraft som håller ihop de tre kvarkarna i en proton eller en neutron. Förmedlingspartikeln kallas gluon, och eftersom kraften har tre "laddningar" som kallas röd, grön och blå, så kallas kraften även färgkraft. Färgkraften är enormt mycket starkare än gravitationen, men den har mycket kort räckvidd. Se frågan nedan för en förklaring till att den ytterst svaga gravitationskraften ändå är den domineramde kraften när det gäller stora objekt som planeter, stärnor och hela universum.

Vad gäller planetbanorna så är en cirkel ett specialfall av en ellips - med storaxeln och lillaxeln exakt lika stora. Det är mycket osannolikt att de i praktiken skall vara EXAKT lika.
/lpe

Se även fråga 957

*

Materiens innersta-Atomer-Kärnor [12124]

Fråga:
Hej, här kommer ett par frågor, som funnits i huvudet ett tag: 1) Plancktid, Planckmassa etc finns, men finns det en Planckladdning? Kvarkar innehåller ju delar av elektronladdningen, så elektronens laddning tycks ju gå att dela, men finns det en minsta laddning? 2) Finns det någon stabil neutral partikel? Elektron resp proton är ju laddade och stabila(?) partiklar medan neutronen ju sönderfaller? Är laddning ett krav för att enb partikel skall vara stabil? 3)Neutrinen, om den är en partikel, påstås kunna byta skepnad på sin färd mellan solen och jorden och borde därför kunna sägas vara instabil, är det en form av 'radioaktivitet'? Finns det instabila och stabila neutriner??
/Thomas Å, Märstagymnasiet, Märsta

Svar:
1 Planckladdning??!! Nej den minsta laddning som man observerat är kvarkars laddning e/3. Det finns inte heller några teoretiska argument för at det skulle finnas mindre laddningar.

2 Det finns stabila neutrala partiklar. Neutrinon till exempel.

3 De s.k. neutrino-oscillationer du refererar till är inte ett sönderfall. Se det hellre så att neutrinon har en komplicerad struktur så den kan visa sig i olika skepnader.
/Gunnar O/Peter E

*

Materiens innersta-Atomer-Kärnor [12275]

Fråga:
Jag har en ganska odefinierad fråga, då jag inte är särskilt insatt i ämnet.

I Geneve finns ju partikelaccelerator, i vilken man efter experiment har kommit väldigt nära Big Bang (ms eller?). Måste man uppnå ett tillstånd med singularitet för att nå ända fram till Big Bang, eller uppnår man det först när man når Big Bang?

Tacksam för svar, gärna på en rimlig nivå.
/Erik N

Svar:
Någon singularitet kan man inte uppnå, men man hoppas åstadkomma tillräckligt hög densitet vid mycket hög temperatur för att kvarkar och glouoner skall bli fria. Detta gör man genom att kollidera tunga atomkärnor med varandra.

Se Kvark-gluon plasma för en rimligt lättförståelig beskrivning av statusen av projektet. CERN-delen av projektet ligger för tillfället nere eftersom man håller på att bygga en ny accelerator (LHC).
/Peter E

*

Materiens innersta-Atomer-Kärnor [12487]

Fråga:
Vad är laddningar (vad är det som gör någonting laddat)? Eftersom en proton (=2u-kvarkar och 1d-kvark) är positivt laddad och en neutron (=1u-kvark och 2d-kvarkar) är neutral, kan man då anta att uppkvarkar är "mer" positiva än ner-kvarkar? (1 uppkvark är lika positiv som två nerkvarkar är negativa?)
/Karl N

Svar:
Uppkvarken u har laddningen +2/3, och nerkvarken d har laddningen -1/3. Protonen uud får då laddningen +1, och neutronen udd laddningen 0. Se Standardmodellen .
/Peter E

*

Materiens innersta-Atomer-Kärnor [12499]

Fråga:
Vad är s.k. "färger" på kvarkar etc.?
/Karl N

Svar:
Kvarkarnas "färger" är inte riktiga färger utan bara en beteckning på en kvarks "laddning". Vanlig laddning finns som + och -, men färgladdning finns i tre varianter, röd, grön och blå. Alla sammansättningar med kvarkar måste vara färglösa eftersom färgkraften är enormt stark. Därför förekommer kvarkar bara som tre kvarkar (baryoner) eller kvark/antikvark (meson).
/Peter E

Se även fråga 1357 och fråga 3931

Nyckelord: färgkraften [8];

*

Partiklar [12617]

Fråga:
Vad är anti-partiklar och vad är dom till för?
/Veckans fråga

Ursprunglig fråga:
Vad är anti-partiklar och vad är dom till för?
/hanna b, kanalskolan, skellefteå

Svar:
All materia i universum är uppbyggd av partiklar. Atomer t.ex. har en kärna av protoner och neutroner som är omgiven av elektroner. Protonerna och neutronerna består i sig av ännu mindre partiklar, s.k. kvarkar. Till varje partikel finns en antipartikel - elektronen har en antielektron (även kallad positron), protonen har en antineutron, osv. Antipartiklarna har samma massa som partikeln men laddningen har motsatt tecken.

Antipartiklar finns normalt inte i fria i naturen, men kan skapas i laboratoriet t.ex. med hjälp av acceleratorer. Man tror dock att i det ögonblick vårt universum skapades i Big Bang (den stora smällen) fanns det lika mycket partiklar som antipartiklar, men på något sätt (som vi ännu inte förstår i detalj) förstördes nästan all antimateria efter en mycket kort tid.

Vad de är till för är svårt att svara på - fysiken sysslar normalt inte med frågan "varför?" utan hellre med "hur?"! Klart är dock att universum är fyllt av symmetrier, och antipartiklarnas egenskaper gör dem till spegelbilder av den vanliga materien.

Läs mer om antipartiklar på Antimatter Academy under länk 1 nedan.
/Margareta H

Se även fråga 124 och fråga 525

Nyckelord: antimateria [16];

1 http://livefromcern.web.cern.ch/livefromcern/antimatter/academy/AM-travel00.html

*

Materiens innersta-Atomer-Kärnor [12713]

Fråga:
Om man skulle samla alla elementara partiklar, eller rättare sagt all massa som existerar på jorden, som inte har något tomrum. DVS, man samlar all massa för sig, och vakum för sig. Alltså, vakum ska inte finnas i det vi samlar ihop. För då går inte våran uppgift ihop som vi fick av våran biologilärare på torsdagseftermiddagen förra veckan. Hur stor skulle den massa bli då? Tror du verkligen på att man skulle kunna hålla den i handen då?

Hoppas du är kunnig nog för att svara på det här nu!
/Christian G, Lidingö Folkhögskola, Stockholm

Svar:
Christian! Frågelådan är ju inte till för att ge färdiga lösningar på problem som getts som räkneuppgifter i skolan (särskilt inte på folkhögskolenivå!), men jag kan ge dig och dina kompisar några ledtrådar...

Jordens materia kan i allra flesta fall anses bestå endast av neutroner, protoner och elektroner, så resten av alla partiklar kan vi lämna därhän. Dessutom är elektronernas massa mycket mindre än nukleonernas (protoner och neutroner kallas med ett ord nukleoner), så dessa kan vi egentligen också bortse ifrån. (Att nukleonerna i sig är uppbyggda av kvarkar och gluoner är inte heller viktigt i detta sammanhang.)

Alltså tolkar jag din fråga så som att vad du egentligen vill komma åt är vilken densitet som ren kärnmateria har - alltså atomkärnornas densitet. I så fall kan man formulera om problemet till: antag att all jordens materia komprimeras till kärnmateriedensitet, hur stor volym får denna då?

Kärnmateria är enormt mycket tätare än den normala materian vi har på jorden. Jordens medeldensitet är ca 5.5*103 kg/m3, medan kärnmateriedensiteten ligger kring 2.3*1017 kg/m3. Till detta kommer att jordens massa är ungefär 6*1024 kg. Med hjälp av dessa uppgifter borde det inte vara så svårt att räkna ut vilken volym kärnmaterie detta motsvarar, och t.ex. vilken radie en sfär med denna volym får. (Jag kan avslöja att den nog blir litet stor för att hålla i den hypotetiska handen!)
/Margareta H

*

Partiklar [12722]

Fråga:
Kvarkar är ju den minsta beståndsdel som upptäckts. Hur mycket vet man om kvarkar? Är en kvark uppbyggd av någonting? Om inte? Vad består den av? Och vilken massa har en kvark? Vad väger en kvark och finns det olika typer av kvarkar beroende på olika former av atomer?
/Christian G, Lidingö Folkhögskola, Stockholm

Svar:
Christian! Jag hoppas att du var nöjd med svaret på din tidigare fråga om jordmassan, och ser med glädje att du nu vill tränga in på djupet av (kärn)materien!

Bra utgångspunkter för att finna svaren på dina frågor är dels att söka på "kvarkar" i vår frågelåda (använd dig gärna av den utmärkta sökfunktionen - se länk till "avancerad sökning" nedan), dels att utforska olika andra webbresurser som t.ex. The Particle Adventure (missa inte att titta på länken till "Particle Chart"), Hands on Cern och A New State of Matter (mest om kvark-gluon-plasma). Dessutom rekommenderar jag varmt Nationalencyklopedin .
/Margareta H

Nyckelord: standardmodellen [24]; kvark [12];

Avancerad sökning på 'kvarkar' i denna databas

*

Materiens innersta-Atomer-Kärnor [12723]

Fråga:
Jag har kollat på dem länkarna, och jag hittade inte det svar jag var ute efter. Dessutom kan jag inte så bra engelska.

Jag vill veta hur stor en kvark är, (vilken volym den har alltså), vad den väger och om det finns olika typer av kvarkar?
/Christian G, Lidingö Folkhögskola, Stockholm

Svar:
Christian! Du ger dig inte, men det är bra ! Om du har problem med engelskan, så kanske det går bättre med norska - du missade nog att The Particle Adventure som jag länkade till i det förra svaret (fråga 12722) även finns i en norsk version. se länk 2 nedan.

Hur stora kvarkar är vet man inte, bara att de är mycket små. Göran Jarlskog som är professor här i Lund skriver i sin utmärkta artikel "En hel värld av nästan ingenting?" (länk 1) att "både elektronerna och kvarkarna har själva en helt försumbar storlek, säkerligen mycket mindre än den experimentella gränsen 10-18 meter."

Man tror att det finns 6 olika kvarkar, som man brukar kalla u (up), d (down), s (strange), c (charm), b (bottom) och t (top). De skiljer sig åt bl.a. genom sina vilomassor och elektriska laddning. Intressant är att kvarkarnas laddningar inte är heltal; så har t.ex u-kvarken laddningen +2e/3 (e är elementarladdningen). De absolut vanligaste kvarkarna är u och d; från dessa är protoner (uud) och neutroner (udd) uppbyggda.

Hur ligger det då till med kvarkars massa? Jarlskog skriver om protonen att de tre kvarkarnas (2 u och en d) sammanlagda vilomassa "bör vara mindre än 20 MeV/c2". (Inom partikelfysiken anger man ofta massa i enheten elektronvolt/c2, eller eV/c2. 1 MeV/c2 (M är prefixet mega) motsvarar då 1.8*10-30 kg.) Detta är mycket mindre än protonens vilomassa som är 938 MeV/c2. "Skillnaden är rörelseenergi som svarar för bindningen av kvarkarna genom utväxlandet av gluoner. Huvuddelen av protonmassan är alltså rörelse." Detsamma gäller för neutronen, som är något tyngre - ca 940 MeV/c2.

Bilden nedan är en kort sammanfattning av kvarkarnas egenskaper.

Fundera vidare: Neutronens vilomassa är större än protonens - vad säger det om vilken av u och d kvarkarna som är tyngst? Kvarkarnas laddningar är alltså fraktioner av enhetsladdningen e - är det unikt, eller känner vi till andra partiklar med samma egenskap?



/Margareta H/Peter E

Se även fråga 12722

Nyckelord: standardmodellen [24]; kvark [12];

1 http://www.hep.lu.se/staff/g.jarlskog//En_varld_av_nastan_ingenting.htm
2 http://www.fys.uio.no/epf/adventures/particleadventure_2.1/index.html

*

Materiens innersta-Atomer-Kärnor [12985]

Fråga:
Hej! Jag har en liten fråga här.. Vid betasönderfall omvandlas en neutron till en proton och en elektron, eller hur? Jag har läst att en neutron består av tre kvarkar, två down(-1/3) och en up(+2/3) som blir laddningen noll. Vid betasönderfallet omvandlas en down-kvark till en up, på så vis blir laddningen +1. Det jag inte förstår är att hur en neutron kan bli en proton OCH en elektron. Varför har neutronen laddningen 0 då, eftersom elektronen har laddningen -1 medan kvarkarna tillsammans är neutrala?
/Helene A, Parkskolan, Mönsterås

Svar:
I kvarkmodellen är neutronens sönderfall en ner-kvark som förvandlas till en upp-kvark och en W- boson:

d(du) --> u(du) + W- --> udu + e- + vanti

(neutronens kvarkbestånd är ddu, protonens är duu)

Observera att laddningen bevaras: -1/3 till vänster blir +2/3 och -1 till höger (om vi betraktar (du) som åskådare). W- bosonen sönderfaller mycket snabbt till en elektron + en antineutrino. På samma sätt kan man se b+-sönderfallet i kvarkmodellen:

u(du) --> d(du) + W+ --> ddu + e+ + v

Så du ser att slutresultatet blir samma som i den klassiska bilden av betasönderfallet.

Se även Betasönderfall#Sönderfallsprocessen .

Nyckelord: betasönderfall [15]; standardmodellen [24]; kvark [12];

*

Materiens innersta-Atomer-Kärnor [13099]

Fråga:
Hej och tack för förra svaret. Ny fråga: I en moderator i ett kärnkraftverk ska neutronhastigheten minskas genom kollisioner med ganska lätta kärnor. Men vad är det för kraft som verkar mellan partiklarna under själva kollisionen? Neutronen har ju tex ingen elektrisk laddning. Den borde åka rakt igenom. Eller är det kanske kvarkarnas laddning som löser problemet?
/Lars J

Svar:
Nej, det är inte kvarkarnas laddning utan kärnkraften. Kärnkraften är inget som skall avskaffas (det som är under diskussion heter kärnenergi), utan kraften mellan nukleoner - protoner och neutroner. Normalt är denna kraft attraktiv (den håller ju ihop atomkärnor), men för mycket små avstånd är den repulsiv (frånstötande). Det är alltså denna ganska komplicerade kraft som får neutronen att växelverka med kärnan den kolliderar med.
/Peter E

*

Universum-Solen-Planeterna [13117]

Fråga:
Varifrån kommer huvuddelen av de atomer som återfinns i en potatis?
/Eva N, Brunn, Ingarö

Svar:
Varför just en potatis? Alla atomer tyngre än helium kommer från processer i stjärnor. Här är en kort sammanfattning:

När universum "skapades" i Big Bang bestod det av energi, kvarkar och leptoner (de fundamentala partiklar som bygger upp all materia). Kvarkarna slog sig ihop till neutroner och protoner.

Inom några minuter bildades även helium från protoner och neutroner, se nedanstående bild som visar förekomsten av lätta kärnor sekunder efter Big Bang. Egentligen bildades protoner (75%) och He-kärnor (25%) eftersom universum fortfarande var mycket varmt - elektronerna var fortsatt fria.

När universum svalnat tillräckligt - eftersom universum expanderar så svalnar det hela tiden - bands elektronerna vid protonerna och He-kärnorna (efter c:a 379000 år, se fråga 11987 ). När temperaturen sjunkit ytterligare bildades stjärnor av en del av gasen.

Först producerar stjärnorna mer helium från vätet i centrum (där det är varmast, 10-20 miljoner grader). När vätet i centrum tar slut ökar temperaturen till c:a 100 miljoner grader och tre heliumkärnor kan slå sig samman till kol-12. Processen fortsätter sedan, och genom infångning av vätekärnor och heliumkärnor bildas alla ämnen upp till järn.

Tyngre kärnor bildas genom att neutroner fångas in. Speciellt vid våldsamma utbrott en stjärna kan få mot slutet av sitt liv - s.k. supernovautbrott - förekommer mycket neutroner. Då bildas alla tunga grundämnen, och alla ämnen spids ut i rymden för att senare kunna ingå i nybildade stjärnor och planeter. Carl Sagan (se Carl_sagan ) uttryckte det poetiskt att vi kommer från stjärnstoft (stardust). Se även Nucleosynthesis , Triple-alpha_process och länk 1 nedan.



/Peter E

Nyckelord: grundämnen, bildandet av [5];

1 http://www.astronomynotes.com/evolutn/s7.htm

*

Materiens innersta-Atomer-Kärnor [13435]

Fråga:
Det verkar konstigt att en taupartikel är en lepton men kan faller sönder till andra partiklar. Leptoner är elementarpartiklar. Som de skriver på "hands-on cern": "de är inte sammansatta av ännu mindre beståndsdelar."
/Aron P, IT-gymnasiet, uppsala

Svar:
Det kan tyckas konstigt, men så är det! Det är inget unikt med tau-partikeln, även kvarkar kan förvandlas till andra kvarkar. Det vanliga betasönderfallet hos en neutron ses i standardmodellen (se elementarpartikel i Nationalencyklopedin ) som nerkvarkens sönderfall till en uppkvark med utsändande av en W-. W-bosonen sönderfaller sedan i en elektron och en antineutrino. Så enligt nuvarande teori behövs inte en inre struktur för att en partikel skall kunna sönderfalla. Det kan emellertid tänkas att denna inre struktur finns, och att vi inte är medvetna om den ännu.
/Peter E

*

Partiklar [13463]

Fråga:
Tja jag är en pojke på 14 vårar som skulle vilja ställa 2 frågor av intresse var vänlig att förklara så tydligt som möjligt med tanke på min ålder: 1. Vad är det för skillnad på fotoner, gluoner och gravitoner (tror jag de hette rätta mig om jag har fel)? 2.Hur uppstår den nukleärakraften, jag vet att den kommer från nukleonerna och jag vet vilka kvarkar de är uppbyggda av men är det bara att acceptera att den finns där. Jag menar laddningar brukar väl inte vilja vara i närheten av samma laddningar?
/Daniel K, björndalen, Trollhättan

Svar:
Daniel! Bra frågor, men naturligtvis inte lätta att svara på. Jag skall försöka göra det så enkelt som möjligt!

1. De partiklar du nämner är förmedlare av (de som överför på avstånd) de grundläggande krafterna. Om vi kompletterar med W,Z så är krafterna och förmedlarna:
  gravitation - graviton (ej påvisad)
  svaga kraften (betasönderfall) - W- och Z0
  elektromagnetiska kraften - foton
  starka kraften (färgkraften) - gluon
Dessa partiklar är inte bara olika i vilka krafter de förmedlar, utan även t.ex. mycket olika tunga. Fotonen har t.ex. vilomassan 0 medan W,Z väger lika mycket som en zirkonium-atom!

2. Du har rätt i att protonerna i kärnan är positivt laddade och repellerar varandra (vill inte vara i närheten av varandra). Det som gör att kärnan hålls ihop är den starka kraften (färgkraften kallas den, inte för att den har något med färg att göra utan för att det finns tre "laddningar) helt enkelt är starkare. De tre kvarkarna som bygger upp neutroner och protoner är "färg"laddade, och det är detta som dels håller ihop tre kvarkar till en neutron/proton, och dels håller ihop neutronerna och protonerna i en atomkärna. Färgkraften var ämnet för årets (2004) nobelpris i Fysik, se länk 1. Exakt hur färgkraften håller ihop atomkärnan har man ännu ingen riktigt bra beskrivning av - det är ett av de hetaste problemen i kärnfysik just nu.
/Peter E

Se även fråga 3456 och fråga 12080

Nyckelord: kärnkrafter [7]; kraftverkningar [9]; färgkraften [8];

1 http://nobelprize.org/physics/laureates/2004/public-sv.html

*

Materiens innersta-Atomer-Kärnor [13536]

Fråga:
tja, undrar när man säger den starka kraften menar man alltid färgkraften inom fysik då? Och vilka partiklar är laddade med vilka färger?
/Karl j, karlsberg, trosa

Svar:
Någon yttring av färgkraften ja. Man kan ibland mena kraften mellan neutroner/protoner i en atomkärna, men ytterst är även denna en yttring av färgkraften. Kvarkar och kraftutbytespartiklarna gluoner har färgladdning. Det finns tre grundfärger rött, grönt och blått (rgb). Kvarkarna har naturligtvis inte färger, de har tre olika laddningar eller egentligen sex, för det finns antifärger också. Endast partiklar som är ofärgade kan förekomma fria: tre kvarkar rgb (t.ex. en proton) eller kvark-antikvark r-anti-r (meson). Allt detta och mycket mer utgör den s.k. Standardmodellen .
/Peter E

*

Materiens innersta-Atomer-Kärnor [13562]

Fråga:
Hejsan. Jag har på flera ställen stött på ordet flytande vacuum. Bland annat i artikeln nedan.

Flera mer fysiskt skolade peroner jag pratat med har avfärdat flytande vacuum som någont som inte kan finnas. Hur fungerar det egentligen? Kan det finnas? Och hur ska jag förklara det för dem för att de ska förstå att det faktiskt finns?
/Kim D, Lernia, Västerås

Svar:
Det finns en referens i artikeln till en annan artikel som är publicerad. I den artikeln har man enligt abstract studerat 'liquid model of the QCD vacuum'. Jag förstår inte mycket av artikeln, men man skall vara försiktig med att tolka uttryck i fysik och speciellt partikelfysik för bokstavligt. Det är tydligen något i standardmodellen (se länken) som säger att vakuum kan ha egenskaper på något sätt analoga med en vätska. Tänk även på att standardmodellen säger att kvarkarna har färger (egentligen färgladdningar), vilket man inte heller få ta alltför bokstavligt.
/Peter E

1 http://www.scielo.br/scielo.php?script=sci_arttext&pid=S0103-97332001000100013&lng=es&nrm=iso

Avancerad sökning på 'standardmodellen' i denna databas

*

Materiens innersta-Atomer-Kärnor [13796]

Fråga:
Hur många olika sorters neutriner finns det?
/Veckans fråga

Ursprunglig fråga:
Hej igen, här är några funderingar som jag fick när jag skrev ett skolarbete i fysiken.

1. Som jag har fattat det så består t.ex. en proton av 3 kvarkar (2 uppkvarkar och 1 nedkvark) men jag har märkt att man säger att den också består av 5% antikvarkar. vad menar man med det, en kvark kan väl inte vara delad på något sätt?

2. Jag läste också att man letar efter Neutriner men är man inte redan säker på att det finns en neutrin i varje partikel familj (1=elektronneutrinon, 2=myonneutrinon och 3=tauneutrinon). Så letar man efter nya neutriner eller vill man bara hitta mer av de som redan är upptäckta för expriment?
/Karl J, Hjärteskolan, Trosa

Svar:
1. Kvarkarna rör sig inne i nukleonerna med mycket höga energier. När de kolliderar bildas kortlivade kvark/antikvark-par. Så man kan alltså säga att nukleonerna till en del består av antikvarkar. Om man säger att de till 5% består av antikvarkar så menar man att under 5% av tiden består nukleonen av tre kvarkar plus ett kvark-antikvark par.

2. Jag vet inte om man kan säga att man letar nya neutrinotyper men däremot neutriner från olika källor. Enligt standardmodellen och mycket detaljerade mätningar av vidden hos Z-partikeln från CERN är det ganska klart att det finns tre typer av neutriner. Figuren nedan visar mätresultatet. Det är klart att 2 och 4 typer av neutriner är inkonsistent med data.

Vad man också är ute efter är att bekräfta att neutrinerna har en vilomassa som är skild från noll. I så fall skulle man kunna få s.k. neutrino-oscillationer, dvs att en neutrino av en typ kan förvandlas till en annan typ. Se vidare Neutrino_oscillation .

Se vidare Standardmodellen och Standard_Model .


Nyckelord: standardmodellen [24]; kvark [12];

*

Materiens innersta-Atomer-Kärnor [14072]

Fråga:
Läste det intressanta svaret på fråga "Partiklar [1424]" om kvarkannihilation, som besvarade några av de funderingar jag själv haft.

En ytterligare undran: I en positiv pi-meson, varför sker det annihilation av en u och en anti-d-kvark? Det är ju inte "spegelpartiklar".

En annan fråga är: Hur kan man mäta "laboratorie-livslängden" på en meson, som ju är i storleksordningen 2 mikrosekunder. Ett vanligt exempel på relativitetsteorins tillämpning är ju de på jorden observerade mesonerna, som inte ska "hinna ner" från sin födelseplats i övre atmosfären. Men om man skapar dem i labbet, får de då lägre (icke-relativistisk) hastighet?
/magnus w, polhemsgymnasiet, göteborg

Svar:
u och anti-d kvarkarna kan inte annihilera eftersom laddningen måste bevaras. Systemet sönderfaller med den svaga växelverkan till en myon och en myon-neutrino.

Pionerna (pi-mesonerna) växelverkar mycket starkt med materia. Med tanke på livstiden du ger menar du nog myoner. Dessa kan man lätt tillverka i labbet och "förvara" med låg hastighet. Det har då den icke-relativistiska livslängden 2 mikrosekunder.
/Peter E

Se även fråga 1424

*

Materiens innersta-Atomer-Kärnor [14216]

Fråga:
Hej, jag undrar en sak som jag skulle vilja få ett svar på. Finns det några speciella egenskaper som identifierar en förmedlings partikel för utom spinnen? T.ex. mesoner kallar man dem förmedlingspartiklar på därför att deras spinn är heltalig? De består ju av kvarkar precis som baryoner...
/Daniel J, Alveskolan, Sundsvall

Svar:
Alla kraftförmedlare har spinn 1, se fråga 3716. Mesoner är som du säger sammansatta av kvark/anti-kvark och är alltså inga fundamentala kraftförmedlingspartiklar. De har emellertid med framgång använts som sådana för att beskriva kärnkraften, alltså kraften mellan protoner och neutroner. Kärnkraften är emellertid en yttring av den starka kraften (som håller ihop kvarkarna i hadroner) på liknande sätt som van der Waalska krafter (länk 1) är en yttring av den elektromagnetiska kraften. Vi har emellertid inte ännu en bra teoretisk beskrivning av detta.
/Peter E

Se även fråga 3716

1 http://www.chemguide.co.uk/atoms/bonding/vdw.html

*

Materiens innersta-Atomer-Kärnor [14224]

Fråga:
Hej, vad menar man när man talar om olika frihetsgrader angående higgsmekanismen? Tack
/Karl L, Bergala, Tynum

Svar:
Hej Karl! Jag är nu på Fysikdagarna i Umeå - ett arrangemang både för universitetslärare/forskare och för gymnasielärare.

Din fråga är inte lätt att besvara - trots att jag hade tillfälle att fråga en partikelfysiker! Teorin om Higgs (en del av den sk Standardmodellen, se nedanstående länk) är ganska abstrakt!

Det bästa jag kan säga är att de fyra frihetsgraderna motsvarar de intermediära bosonernas (W+, W- och Z0) massor samt higgspartikelns massa.

Övriga partiklar (leptoner, kvarkar) får sin massa på annat sätt.
/Peter E

Nyckelord: standardmodellen [24];

*

Materiens innersta-Atomer-Kärnor [14238]

Fråga:
Relativt nyligen läste jag i FoF om preoner. Vad är det, vad tror man är "byggstenarna"? Ligger det på strängteorinivå, med gömda dimesioner? Preoner finns inte med i NE, däremot hittade jag en amerikansk artikel från -97 där ordet förekom.
/Rolf B, Hallägraskolan, Halmstad

Svar:
Länk 1 är en sammanfattning av artikeln i Forskning & Framsteg 7/05. Den spekulerar om kompakta objekt som skulle kunna finnas om preoner existerar. Lite väl spekulativt för min smak !

Preonteorin (länk 2) är en teori från 80-talet att kvarkar och leptoner skulle vara uppbyggda av preoner. Något experimentellt stöd för teorin har inte kommit fram, så teorin har minskat i popularitet.
/Peter E

1 http://www.fof.se/?id=05740
2 http://en.wikipedia.org/wiki/Preon

*

Materiens innersta-Atomer-Kärnor [14516]

Fråga:
Hej, nu har jag läst lite QCD som är otroligt intressant men en fråga uppstår. jag finner aldrig något svar varför det bara finns kvarksystem med tvåkvarkar(mesoner) eller trekvarkar(baryoner)? skulle inte en partikel med 10kvarkar vara möjlig den skulle väl också kunna vara neutral i "färgen"?
/Karl J, Hjärteskolan, trosa

Svar:
Karl! Färg-antifärg och tre olika färger är de enklaste färglösa systemen. Alla kombinationer av dessa är också färglösa. Man har letat efter dem, men såvitt jag vet inte hittat någon. Indikationer på ett fyrkvarksystem för några år sedan var tydligen falskt alarm.
/Peter E

*

Materiens innersta-Atomer-Kärnor [14707]

Fråga:
Hej Lund! Jag har en fråga gällande elementarpartiklar. I en lärobok för fysik B i gymnasiet stod det att man tror att kvarkar är uppbyggda av något som man valt att kalla rishoner, en annan uppfattning är strängteorin. Jag antar att strängteorin är mer utbredd i dagens läge fast hur långt har man egentligen kommit på "rishonmodellen"?
/Oscar S, Östanå gymnasium, Eksjö

Svar:
Oscar! Jag har aldrig hört talas om rishoner, och jag tycker det är mycket konstigt om en så lite etablerad teori kommer in i en gymnasielärobok. Kvarkar är enligt standardmodellen elementarpariklar. Strängteori n är ännu inte i ett stadium där den kan förutsäga något.
/Peter E

*

Materiens innersta-Atomer-Kärnor [15147]

Fråga:
Finns det mindre partiklar än elektroner??
/alexander b, skytteholmsskola, solna

Svar:
Alexander! Det beror på vad man menar med liten. Alla elementarpartiklar (leptoner och kvarkar, se fråga 9598) anses vara punktformiga, så i den meningen är alla lika stora. Om man med storlek menar chansen att kollidera med andra partiklar så är neutrinen minst.
/Peter E

*

Materiens innersta-Atomer-Kärnor [15152]

Fråga:
mesoner, är de budbärare eller vad, vilken kraft?

varför har svaga kraften så omständigt namn (inter mediär vektor boson) jämfört med de andra?
/alf e, acheberg, göteborg

Svar:
Mesoner används för att beskriva kraften mellan nukleoner i en kärna. Detta är inte en fundamental kraft (och mesonerna inte kraftförmedlingspartiklar enligt standardmodellen ) utan en yttring av den starka färgkraften som håller ihop kvarkar. Se fråga 14216 och länk 1 nedan.

Partiklarna kallas vektorboson för att spinnet är 1 (som alla kraftförmedlarpartiklar). Intermediär vet jag inte var det kommer ifrån. Partiklarna kallas numera helt enkelt W+, W- och Z0. Se länk 2.
/Peter E

Se även fråga 14216

Nyckelord: standardmodellen [24];

1 http://hands-on-cern.physto.se/hoc_v21sv/rollover/ro_pimesonkraft.html
2 http://hyperphysics.phy-astr.gsu.edu/hbase/particles/expar.html#c3

*

Partiklar [15154]

Fråga:
Om gluoner
/Veckans fråga

Ursprunglig fråga:
svårt hålla isär: mesoner växelverkar alltså mellan protoner/neutroner, medan gluoner växelverkar mellan kvarkar. Om det finns 8 olika gluoner, så hur skiljer man dem, vad har de för beteckning, tecken? Mvh
/alf e, ascheberg, gbg

Svar:
Alf! Det finns 8 olika gluoner. Alla med färg/antifärg kombinationer. Eftersom det finns tre färger - rött, grönt, blått - tycker man att det skulle finnas 9 gluoner. Av subtila skäl som är ganska svårt att förstå finns det i själva verket bara 8 gluoner, se Gluon och länk 1.

Den speciella kombinationen som faller bort är

(röd-antiröd + grön-antigrön + blå-antiblå)/sqrt(3)

Anledningen är att denna skulle kunna växelverka med vilken färg som helst utan att ändra egenskaper. Någon sådan växelverkan har inte observerats, så denna kombination faller bort.

Lite förenklat fungerar utbytet av gluoner som i nedanstående tabell. Vi utgår från en röd och en grön kvark (1). Den gröna kvarken sänder ut en grön/antiröd gluon och blir röd (2). Den ursprungliga röda kvarken absorberar gluonen och blir grön (3). Kvarkarna har alltså bytt färger och det är detta som ger attraktionen.

1 R   G
.
2 R   granti   R
.
3 G   R 

/Peter E

Nyckelord: standardmodellen [24]; färgkraften [8]; gluoner [7];

1 http://math.ucr.edu/home/baez/physics/ParticleAndNuclear/gluons.html

*

Materiens innersta-Atomer-Kärnor [15482]

Fråga:
Vilka är de viktigaste/mest kända olösta problemen inom kärnfysik?
/Eir L, Umeå, Umeå

Svar:
Eir! Åsikten om vilket problem som är viktigast beror på vilken kärnfysiker du frågar. När det gäller grundläggande kärnfysik skulle jag vilja se en beskrivning av atomkärnan i termer av kvarkar och den fundamentala färgkraften. Nuvarande beskrivning använder sig av att kärnan består av neutroner och protoner (nukleoner) som hålls ihop av en mystisk och mycket komplicerad kraft.

I själva verket består ju nukleonerna av tre kvarkar som hålls ihop av den starka färgkraften. Nukleonerna i atomkärnan hålls sedan ihop med någon sorts "restväxelverkan" mellan de olikfärgade kvarkarna i nukleonerna. En detaljerad beskrivning av detta saknas i dag.

Trots denna avsaknad av en heltäckande modell tycker jag att vi förstår en hel del av atomkärnans egenskaper. T.o.m. den enklaste modellen, vätskedroppsmodellen (se Liquid_drop_model ), beskriver många fenomen på ett utmärkt sätt. Sedan finns det mer komplexa modeller som t.ex. skalmodeller (Shell_model ) och kollektiva modeller som väl beskriver många egenskaper hos atomkärnor.

Det är emellertid klart att vår beskrivning av atomkärnan inte är på långa vägar så exakt som beskrivningen av atomen, dvs elektronernas rörelser kring atomkärnan. Detta beror dels på att i atomfallet finns en tung laddad kärna som ger den elektriska potential som elektronerna rör sig i. I atomkärnan bildar nukleonerna själva den potential de rör sig i. En annan anledning till att atomkärnan är mer svårbeskriven är att egenskaperna hos kraften mellan nukleonerna är mycket komplexa - kraften beror förutom av avståndet mellan två nukleoner även av relativa spinnet, vinkeln mellan nukleonerna och spinnet (kraften är vad man kallar icke-central), hastigheten och eventuellt om det finns en tredje nukleon i närheten.
/Peter E

Nyckelord: kärnfysik [2]; färgkraften [8];

*

Materiens innersta-Atomer-Kärnor [15771]

Fråga:
Nu när den stora acceleratorn LHC startar på CERN: är det risk att det bildas ett svart hål som hela jorden kan försvinna i?
/Veckans fråga

Ursprunglig fråga:
Nu när den stora acceleratorn LHC startar på CERN: är det risk att det bildas ett svart hål som hela jorden kan försvinna i?
/Mamma till orolig lågstadieelev

Svar:
Nej, det är ingen fara! Det bildas inga svarta hål!

Förhoppningsvis kan man genom kollisioner åstadkomma materia med mycket hög temperatur och densitet, och denna skulle kunna ha andra egenskaper än vanlig materia - den skulle kunna tänkas innehålla fria kvarkar. Detta kallas kvark-gluon plasma. Men som sagt, det är helt ofarligt. Naturen utför ännu mer våldsamma experiment när kosmisk strålning träffar jorden. Det förekommer kärnpartiklar med energier upp till 1 joule i den kosmiska strålningen - detta är mycket mer än vad man kan åstadkomma med LHC.

Det andra man vill åstadkomma med LHC är att hitta den s.k. higgspartikeln . Denna behövs för den s.k. standardmodellen .

Länk 1 nedan är en artikel om LHC från Sydsvenskan. Mer information finns på LHC och CERN . Den senare har en artikel om säkerheten med LHC där det bland annat står:

“The LHC will enable us to study in detail what nature is doing all around us,” said CERN Director General Robert Aymar. “The LHC is safe, and any suggestion that it might present a risk is pure fiction.”

Tillägg 11 september 2008:
Det har på andra sätt kommit in en del frågor om farligheten hos LHC. Delvis baseras detta på information i en liten artikel av Johan Hakelius i Aftonbladet. Han är dock tydligt ironisk mot uppgifterna, och han kan ju inte hjälpa att några tossar förutspår världens undergång. Det har alltid funnits sådana, se t.ex. The_End_Is_Nigh#Origin_of_name .

Anton har en annan synpunkt förutom jordens undergång: skall man lägga ner miljarder euro för att bygga en accelerator som kanske kommer fram till något som knappast är relevant för de flesta människor? Ja, så länge man har en hygglig balans mellan direkt nyttig forskning och nyfikenhetsforskning så tycker jag det! Hade människan inte varit nyfiken så hade vi varit kvar i träddungarna i Afrika!

Många av de nyttigheter vi har idag (och för all del även onyttigheter) är direkta resultat av nyfikenhetsforskning. Ett exempel från CERN är World Wide Web. I dag är webben en vida spridd och viktig nyttighet: från början var det ett sätt för forskarna att kommunicera sina resultat tills Tim_Berners_Lee kom på att kanske hela värden behöver kommunicera!

Det viktigaste skälet för nyfikenhetsforskning är emellertid att människan är inte människa om hon inte får ägna sig åt annat än det direkt för överlevnaden nyttiga! Det är ju ändå så att en symfoni av Beethoven, en vacker tavla och en liten förståelse hur vi passar in i universum gör livet rikare!

Tack Madelene för Aftonbladet-artikeln och Nils-Göran för Antons inlägg!

Två intressanta videor:


/Peter E

Nyckelord: higgspartikeln [10]; standardmodellen [24]; gluoner [7];

1 http://sydsvenskan.se/varlden/article365220/De-storsta-svaren-finns-i-de-minsta-bitarna.html

*

Materiens innersta-Atomer-Kärnor [15922]

Fråga:
Materia och antimateria trivs ju inte ihop utan annihileras till strålning. Mesoner består av en kvark och en antikvark. Men kan de då alls existera?
/Veckans fråga

Ursprunglig fråga:
Hej! Materia och antimateria trivs ju inte ihop utan annihileras till strålning. Mesoner består av en kvark och en antikvark. Men kan de då alls existera? Eller är anti i antikvarkar av annan innebörd än anti i antimateria? Annihilerar även elektronneutriner och deras antineutriner?
/Thomas Å, Arlandagymnasiet, Märsta

Svar:
Ja det kan tyckas konstigt att antipartiklar inte annihilerar varandra, men det finns andra saker som måste bevaras, t.ex. laddning. Låt oss titta på den först upptäckta mesonen, p-mesonen eller pionen.

Den neutrala pionen består som synes i nedanstående figur av en up-kvark och en anti-up-kvark eller en ner-kvark och en anti-ner kvark (i själva verket är pionen en kombination av dessa). Dessa kan utan problem annihilera precis som en elektron och en positron. Kvar blir bara två fotoner med hög energi. Eftersom det är en elektromagnetisk process går den mycket snabbt - medellivslängden för p0 är 10-16 sekunder.

De laddade pionerna är kombinationer av en kvark och en anti-kvark av en annan typ. Den negativa pionen består av en ner-kvark och en anti-upp-kvark. Denna kombination kan inte annihilera eftersom kvark och anti-kvark är av olika typ. Dessutom kan det inte bli bara strålning kvar eftersom laddningen måste bevaras. Den negativa pionen måste därför sönderfalla med den svaga växelverkan via den intermediära bosonen W-. Detta tar mycket längre tid, och jämfört med p0 är p- "nästan stabil" med en livslängd på 10-8 sekunder. Se vidare Pion .

Såvitt jag förstår kan elektronneutriner annihilera med anti-elektronneutriner men sannolikheten att de skall växelverka är mycket liten.



/Peter E

Se även fråga 1424

Nyckelord: annihilation [14]; antimateria [16]; kvark [12];

*

Materiens innersta-Atomer-Kärnor [15955]

Fråga:
Hej! En foton kan ju spridas mot en elektron och därvid ändra riktning och energi(frekvens el våglängd). Bör den därvid anses vara en ny/annan foton än envar före studsen/spridningen? Man lär ju inte kunna dela på fotoner, men vid spridning tycks det få att ändra på dem i alla fall! När Jag ser mig i spegeln, så måste ju en foton vid studsen överföra rörelsemängd till spegeln. Ändras därvid fotonens energi också? (Och är det samma foton" som återkommer till mig?) Innehåller rödförskjutna fotoner samma energimängd som icke-rödförskjutna eller har det skett en "adiabatisk avkylning(!)" av dem?
/Thomas Å, Arlandagymnasiet, Märsta

Svar:
Thomas! Eftersom fotoner (och även kvarkar och leptoner) är identiska partiklar saknar det mening fundera på om det är samma eller en annan foton.

Ja, rörelsemängden som överförs till spegeln tas från fotonens energi. Den spridda fotonens energi är allså lite lägre än den ursprungliga. För en spegel som med stor massa är effekten mycket liten, men för spidning av en högenergetisk foton på en elektron (comptonspridning ) är effekten stor.
/Peter E

*

Materiens innersta-Atomer-Kärnor [16151]

Fråga:
1 Jag läste i en artikel om Kvark Gluon Plasma (som enligt någon var samma sak som ett svart hål(?))

2 Och läste att man försöker framställa det i partikelacceleratorn LHC genom att krocka tunga joner, finns det en möjlighet att ens framställa ett svart hål i denna (läste att svarta hål endast kan exstera efter en stjärnsmäll med en stjärna större än 3 solmassor)??

3 Och sker kollisioner mellan tunga joner naturligt i univerum i liknande hastigheter?

4 Läste även om svarta minihål, existerar dessa, och hur tror man att de bildas? och skulle dessa kunna sluka jorden?

5 Denna Kvark-gluon plasma som man förutspår ska bildas vid höga tempraturer, kan den överleva när det svalnar eller faller sen sönder i och med att det. Eller behövs den höga tempraturen för att plasman ska behållas och inte försvinna?
/Kristina B, Fredrika Bremer, Nynäshamn

Svar:
1 Nej, kvark-gluonplasma, se fråga 4220 nedan, är bara fria kvarkar och gluoner och har inget med svarta hål att göra. Se även fråga 15771 även om denna behandlar ett annat experiment.

2 Nej, se fråga 15771.

3 Ja, tom med högre energier än man kan åstadkomma med LHC, se Large_Hadron_Collider . Energin kan vara så hög som 1020 eV, dvs över en joule!

4 Det har spekulerats om att dessa skulle har skapats vid big bang. De bör emellertid ha försvunnit nu, och de har aldrig observerats. Se vidare länk nedan om hawkinstrålning.

5 Nej, den försvinner snabbt och blir till vanlig materia eftersom plasman expanderar och kyls.
/Peter E

Se även fråga 4220 och fråga 15771

Avancerad sökning på 'hawkin strålning' i denna databas

*

Materiens innersta-Atomer-Kärnor [16236]

Fråga:
Hur har atomens nya uppbyggnad med kvarkar och strängar påverkat vårt sätt att se på vår omvärld? Tack i förhand!
/sofie b, göksten, eskilstuna

Svar:
Sofie! Inte ett dugg skulle jag säga! Strängteorin har ännu inga konsekvenser i observationer så den lever lite sitt eget liv.

Inte heller kvarkteorin (standardmodellen), som däremot är mycket väl etablerad, har några praktiska konsekvenser för kärnfysik och atomfysik. Man räknar fortfarande på atomkärnan som en samling neutroner/protoner. Först när man går till högre energier och studerar hadroner (protoner, neutroner, mesoner mm) kommer kvarkar och gluoner in i bilden.

Länk 1 är en kul sång, Bohemian Gravity (originalet Bohemian Rapsody av Queen), som bland annat handlar om strängteori.
/Peter E

Nyckelord: strängteori [7]; standardmodellen [24];

1 http://www.youtube.com/watch?v=2rjbtsX7twc&feature=youtu.be

*

Materiens innersta-Atomer-Kärnor [16404]

Fråga:
Hej! Några sönderfallsfrågor: Enligt en tabell är protonen inte säkert stabil, utana/men har en medellivslängd >_10^31 år. Universums nuvarande ålder är ju rätt mycket mindre än de tildslängden så någon större mängd sönderfall har väl knappast registrerats. Men vilka är de tänkta sönderfallsprodukterna? En partikel "delta" har medellivslängden ~10^-23 s, (finns begreppet naturlig kärntid än?) men saknar angivna sönderfallsprodukter. Är de okända eller blir de "bara energi" dvs strålning? Vart tar deras laddning i så fall vägen? Vilomassan för en fri kvark är okänd, men skulle en fri kvark vara stabil? Tauneutrinens stabilitet tycks ha varit ifrågasatt. Gäller detta än eller är den stabil numera?
/Thomas Å, Arlandagymnasiet, Märsta

Svar:
Thomas! Du vet vad jag tycker om flerdelade frågor !

Vid eventuellt brott mot baryontalets bevarande kan protonen sönderfalla så här:

p -> e+ + p0

se Proton_decay . Sönderfallet har ej observerats.

Delta-resonansen sönderfallet till en nukleon och en pion, se Delta_resonance . Laddningen måste bevaras.

Eftersom det inte finns fria kvarkar kan man inte säga något om stabiliteten. Antagligen skulle bara den lättaste kvarken vara stabil.

Enligt standardmodellen är tau-neutrinen stabil. Eftersom neutrinerna ganska säkert har massa, så kan man pga neutrino-oscillationer hävda att alla neutriner är instabila
/Peter E

*

Blandat [17269]

Fråga:
Hej! Jag såg ett program på TV för ett tag sedan som handlade om fraktaler - självlikformighet. Där påstod de att "Allt består av fraktaler", "Allt från människokroppen till bergstoppar" Det uttrycket fick mig att fundera lite.

Det är ju så att elektroner kretsar runt atomkärnan. Om man zoomar ut ganska kraftigt så märker vi att månen kretsar runt jorden, likt elektronerna runt atomkärnan. Jorden i sin tur kretsar runt solen som själv kretsar runt vintergatans mitt. Rörelsen är likformig på olika nivåer, eller hur?

Men om det skulle vara en regel så måste förstås galxerna kretsa runt någonting? Vad är då detta "något"?

Har dessa fenomen något att göra med gravitation, i så fall vad?

Om detta stämmer så borde krökningen i rummet som galaxerna orsakar, också påverka oss på något sätt, stämmer det? hur?
/Marcus B, Östlyckeskolan, Alingsås

Svar:
Marcus! Det är sant att naturen är uppbyggd av system av ökande storlek - från kvarkar som bygger upp nukleoner, nukleoner som bygger upp atomkärnor ... till galaxhopar och superhopar. Men systemen är inte särskilt lika. Speciellt tycker jag att Bohrs atommodell med elektroner som cirklar kring en kärna inte beskriver verkligheten se fråga 14369 . Dessutom är det för de atomära och subatomära systemen coulombkraften och den starka färgkraften som verkar men för de större systemen är det gravitationskraften.

Galaxer i en galaxhop kretsar runt den gemensamma tyngdpunkten - summan av påverkan från alla galaxer och eventuell mörk materia. De påverkas dessutom av närbelägna galaxer. Kraften är alltså gravitationskraften.

Krökningen av rummet påverkar oss inte direkt - vi är ju inne i det. Vi kan emellertid se krökningen av rummet pga galaxhopar, se fråga 1400 .
/Peter E

*

Universum-Solen-Planeterna [17472]

Fråga:
Hej! Diametern på det synliga universum har angivits till 93 miljarder ljusår. Universums ålder anges till c:a 13,8 miljarer år. Om universum utvidgats med ljushastigheten borde storleken väl vara c:a 13 miljarder ljuår "i radie", dvs det dubbla i diameter. Diskrepansen är rätt stor. är orsakentill den den s k inflationen, som anses ha skett strax efter begynnelsen? Hur länge pågick inflationen, vad orsakade den (=hur kom den till) och vilken var dess hastighet -den tycks ju ha gått fortare än ljuset!-?
/Thomas Å, Knivsta

Svar:
Hej Thomas! Var har du fått 93 miljarder år från (se dock Observable_universe#Size )? Om universum är 14 miljarder år gammalt så är diametern hos det synliga universum definitionsvis 28 miljarder ljusår. Hur stort "hela" universum är vet vi helt enkelt inte, det kan vara oändligt stort. Se fråga 6116 .

Inflation är att universum under någon bråkdels sekund nästan direkt efter big bang expanderade extremt fort - storleken beräknas ha ökat ungefär 1028 gånger, se Inflation_(cosmology) .

Expansionen vid inflationen var mycket kortvarig omkring 10-38 s.

Inflationsperioden var mycket kortvarig från 10−36 sekunder efter Big Bang till mellan 10−33 och 10−32 sekunder efter Big Bang.

Inflationen skedde med en hastighet som översteg ljushastigheten. Orsaken kan vara en fasförändring (som frigjorde energi) då den starka (färg)kraften skildes från den elektrosvaga, se fråga 1496 . Se länk 1. De flesta kosmologer anser att inflationen som sådan är väl etablerad. Tidpunkterna är emellertid osäkra - vi har inga observationer från så nära Big Bang. Det finns förhoppningar att detta kan ändras med utvecklandet av mer känsliga gravitationsvågsdetektorer.

Bilden nedan av de fyra olika kraftverkningarna är från länk 1. I standardmodellen finns fyra kraftverkningar:

* gravitation
* svag växelverkan (betasönderfall)
* elektromagnetism
* starka färgkraften (QCD - håller ihop kvarkar)

Dessa kraftverkningar var vid big bang förenade i en kraft. Efter hand har ur-kraften separerats till fyra olika krafter. Kärnkraften (som håller ihop protoner och neutroner i atomkärnan) är inte en separat kraft utan en yttring av den starka färgkraften.



/Peter E

Nyckelord: inflation [7]; big bang [37]; standardmodellen [24]; kraftverkningar [9]; gravitationsvågor [19];

1 http://www.daviddarling.info/encyclopedia/B/Big_Bang.html
2 http://aether.lbl.gov/www/science/inflation-beginners.html

*

Partiklar [17502]

Fråga:
Antimateria på CERN
/Veckans fråga

Ursprunglig fråga:
Hej! I CERN har det producerats antimateria, enligt tidningsrapporter. Har antimateriepartiklarna samma kvanttal som vanlig materia? Följer antimaterian Paulipricipen? Hur skiljer/påvisar man en neutron från en antineutron, de är ju båda oladdade?
/Thomas Å, Knivsta

Svar:
Thomas! Det nya är att man lyckats "klä på" antiprotoner med positroner och alltså lyckats framställa ett litet antal (38) anti-väteatomer. Det innebär att man kan studera övergångar i anti-väte och jämföra dem med väte. Standardmodellen säger att egenskaperna skall vara exakt desamma med undantag för laddningen. Antipartiklar följer Pauliprincipen mot andra identiska antipartiklar, mot motsvarande partiklar saknar Pauliprincipen mening.

Man alltså lyckats framställa en liten mängd oladdad antimateria. Än så länge inte tillräckligt för att scenariot i Dan Browns bok (och filmen) Änglar och demoner skall bli verklighet!

Neutronen har t.ex. ett magnetiskt moment. Anti-neutronen har det motsatta eftersom kvarkarna har annan laddning. Neutronen har kvarksammansättningen ddu (laddning -1/3,-1/3,+2/3). Antineutronen har sammansättningen dantidantiuanti (laddning +1/3,+1/3,-2/3).

Se vidare "over the top" artikeln länk 1 och den mer sansade pressreleasen länk 2.
/Peter E

Nyckelord: standardmodellen [24]; antimateria [16]; kvark [12];

1 http://www.theregister.co.uk/2010/11/18/cern_antimatter_bomb/
2 http://press.web.cern.ch/press/PressReleases/Releases2010/PR22.10E.html

*

Partiklar [17620]

Fråga:
Hej! Kallas kvarkar för elementarpartiklar? Gör neutroner och protoner det fortfarande fast de består av mindre partiklar i form av kvarkar?
/Sarah A, Lillerudsgymnasiet, Vålberg

Svar:
Sarah! Ja, kvarkar är elementarpartiklar enligt standardmodellen . Nej, protoner och neutroner kallas inte elementarpartiklar i dag eftersom de är sammansatta av kvarkar.

Figuren i fråga 9598 visar alla elementarpartiklar enligt standardmodellen.
/Peter E

*

Materiens innersta-Atomer-Kärnor [17763]

Fråga:
Hej! I en länk till en länk i ett frågesvar fanns en tabell där gluoner skrevs ha vilomassan = 0. Det finns ju begreppet kvark-gluonplasma, där hög fart väl är rådande (det är ju hett) men eljest förknippar jag gluonerna med atomkärnan och den är rätt liten. Innebär masslösheten då att de alltid skulle röra sig med ljushastigheten? Även inuti atomkärnan?

Vilomasslösa partiklar syns vara fotonen, gravitonen och gluonen. De skall då röra sig med ljushastigheten, c. Fotonens energi/egenskaper bestäms av dess frekvens. Vilka egenskaper bestämmer ev energi/egenskaper hos de andra två förmedlarpartiklarna?
/Thomas Å, Knivsta

Svar:
Thomas! En länk till en länk! "Your honour I protest, this is hearsay!" Det finns emellertid en förstahandskälla: Gluon . Vilomassan för gluonen är 0 enligt standardmodellen.

Ja, om gluonerna är masslösa har de alltid hastigheten c. Normalt är kvarkarna och gluonerna (som håller ihop kvarkarna med vad som kallas den starka färgkraften, se nedanstående länkar) inneslutna i utåt färglösa system. Vid mycket hög energi (temperatur) kan man tänka sig att man får en soppa med fria kvarkar och gluoner. Det är detta som kallas kvark-gluon plasma, se Quark–gluon_plasma .

Van gäller vilomasslösa partiklar, se fråga 13912 . Det är alltså våglängden hos materievågorna som ger den totala energin.
/Peter E

Nyckelord: standardmodellen [24]; färgkraften [8]; gluoner [7];

*

Elektricitet-Magnetism [18184]

Fråga:
Vad är egentligen laddning och vad orsakar laddning? Har t.ex elektronen en laddning p.g.a sitt spinn? Eller har t.ex protonen en positiv laddning p.g.a av sin uppbyggnad av kvarkar som i sin tur har spinn? Hur hänger det egentligen ihop?

Tack på förhand!!
/Anna P

Svar:
Elektrisk laddning är den fysikaliska storhet som påverkas av elektromagnetisk växelverkan, en av de fyra fundamentala krafterna. Laddningen motsvarar massan för gravitationskraften.

Nej, laddning har inget med spinn att göra, men en laddad partikel med spinn (elektron eller proton) får ett magnetiskt moment. Eftersom protonen består av laddade kvarkar så får den sin laddning från dessa.

Se vidare Elektrisk_laddning .
/Peter E

*

Materiens innersta-Atomer-Kärnor [18577]

Fråga:
Såvitt jag har förstått det hela rätt så hålls atomkärnan i huvudsak ihop av "stark växelverkan". I grund och botten så är det inte den "egentliga" starka växelverkan, utan snarare "pioner" som uppkommer som en bieffekt. Varför skapas de här pionerna, och vilken effekt har de på de protoner och neutroner som växelverkar? Vad består de av?

Den "riktiga växelverkan" sker då "inuti" nukleonerna, med gluoner som fältpartikel. Men gluoner har också en färgladdning, precis som kvarkarna. Vad har det här för effekt? Byter kvarkarna färg?

Dessutom så ska det krävas som minst gluoner om atomkärnan har ett visst antal nukleoner i förhållande till protoner (tror att det är Järn med 30 neutroner som är stabilast), och då "frigörs" gluoner om t.ex atomkärnor med mindre antal neukloner slår sig ihop för att komma närmare det här talet. Varför?
/Axel H, Tunaskolan, Lund

Svar:
Axel! Bra och svåra frågor du ställer! Det är saker som teoretiker jobbar med att förstå!

Vad gäller beskrivning av kärnkraften se fråga 1720 . Det är korrekt att den starka kraften orsakas av att kvarkarna är färgade.

Gluonernas egenskaper och påverkan på kvarkarna behandlas i fråga 15154 . Som du ser har gluonerna färg,anti-färg och får kvarkarna att byta färg.

Vilka kärnor som blir stabila och hur stabila kärnor sönderfaller behandlas i fråga 13758 .
/Peter E

Nyckelord: kärnkrafter [7]; färgkraften [8];

*

Partiklar [18674]

Fråga:
Hej! I NyTeknik idag nämns en ny kvasipartikel, orbitonen, samt ett par stycken redan kända, holon och spinon, alla kopplade till elektronen. Trots litet googlande, undrar jag om det är fråga om partiklar "i sig", jämförbara med kvarkar, eller vad annars, ty en elektron anses ju odelbar.
/Thomas Å, Knivsta

Svar:
Se länk 1 för NyTeknik artikeln och länk 2 för en lite mer avancerad artikel från Nature.

Nej, det är inte vanliga partiklar som elektroner och kvarkar. Kvasipartiklar är en teoretisk konstruktion som har med vågfunktionen att göra. I ett sammansatt system men flera atomer och elektroner kan olika aspekter på elektronens vågfunktion tänkas hamna på olika ställen. Det är emellertid inte helt lätt att förstå.
/Peter E

1 http://www.nyteknik.se/nyheter/innovation/forskning_utveckling/article3460971.ece
2 http://www.nature.com/news/not-quite-so-elementary-my-dear-electron-1.10471

*

Partiklar [18813]

Fråga:
jag har länge undrat över om det är någon stor chans att vi människor kommer hitta ännu mindre beståndsdelar än kvarkar eller om det är bortom våra förmågor?
/isabel s, kunskapsskolan, spånga

Svar:
De flesta tror nog att kvarkar och leptoner inte är sammansatta av andra partiklar. Däremot kan kanske vår syn på kvarkar ändras t.ex. genom strängteorin, se fråga 17334 .
/Peter E

*

Blandat [18849]

Fråga:
Hur har upptäckten av standardmodellen påverkat människan?
/Veckans fråga

Ursprunglig fråga:
Hejsan! Hur har upptäckten av standardmodellen påverkat människan? Vad är de positiva resp. negativa effekterna av upptäckten?
/Agnes H, Donnergymnasiet, Göteborg

Svar:
Agnes! Med standardmodellen inom partikelfysik avses den modell som beskriver de minsta partiklarna och deras interaktioner (kraftverkningar) genom elektromagnetisk, stark och svag växelverkan med hjälp av kvantfältteori. Standardmodellen är en kvantmekanisk teori, men den är ingen heltäckande modell eftersom den inte innefattar gravitationskraften. Se bilden nedan och Standardmodellen .

Standardmodellen kom inte till genom att en person hade en bra idé. Den kom till genom att man under 50-60-talen med hjälp av allt större acceleratorer upptäckte fler och fler partiklar. Till sist blev det en förfärlig röra med partiklar. Då upptäckte några fysiker ett mönster i partiklarnas egenskaper genom att klassificera partiklarna med avseende på olika egenskaper. Man kunde då se att vissa platser i schemat var tomma. Då fick man en antydan till vad man skulle leta efter. Nu har man hittat alla partiklar i det ursprungliga schemat. Om det finns fler partiklar än de som omfattas av standardmodellen vet man inte.

I standardmodellen ingår två typer av partiklar, leptoner och kvarkar. Dessutom ingår higgspartikeln (som har att göra med elementarpartiklarnas massor) och kraftförmedlingspartiklar.

Lepton: Leptonerna är den ena huvudgruppen av fermioniska (med halvtaligt spinn) elementarpartiklar. Av leptoner finns det sex stycken aromer, vilka liksom kvarkarna delas in i tre familjer. Varje familj består av en partikel och tillhörande neutrino. Se Lepton .

En kvark är en elementarpartikel som tillsammans med en eller flera andra kvarkar bygger upp den grupp partiklar som kallas hadroner (t.ex. nukleoner och mesoner). Så vitt man vet idag är kvarkarna, tillsammans med leptonerna, materiens minsta byggstenar. Det finns sex olika typer av kvarkar, kända som aromer. Aromerna med den lägsta massan, uppkvarken och nedkvarken, är i allmänhet stabila och mycket vanligt förekommande i universum. Se Quark .

Den viktiga skillnaden mellan leptoner och kvarkar är att leptoner växelverkar bara med den svaga kraften medan kvarkar växelverkar med både den svaga och starka kraften.

Här är en kul föreläsning om hur atomer och kvarkar är uppbyggda:

Eftersom mycket få personer fullt ut förstår standardmodellen är dess praktiska betydelse mycket begränsad. Det ligger emellertid i människans natur att försöka förstå världen omkring sig, och för detta är standardmodellen tillsammans med big bang teorin mycket viktiga puzzelbitar. Även om man inte förstår teorierna fullständigt, kan det vär vara intressant att veta att universum är uppbyggt av kvarkar och leptoner och att universum skapades i en gigantisk explosion för 13.7 miljarder år sedan...?

Ett annat skäl att bedriva avancerad forskning t.ex. på CERN är att man utvecklar ny teknik som kan användas för andra ändamål. Ett exempel är avbildning för medicinsk diagnos (PET, se Positron_emission_tomography ).

Ett annan utveckling är när Tim Berners-Lee (Tim_Berners-Lee ) på CERN ville åstadkomma ett informationssystem som forskarna kunde använda för snabb kommunikation av data och idéer. Det dröjde inte länge innan man insåg att detta system hade generell tillämpbarhet, och WWW (World_Wide_Web ) var fött.

Avancerad forskning är även utmärkt för att utbilda nästa generations forskare, lärare, industriledare och entreprenörer.

Några negativa effekter av grundläggande forskning är svårare att hitta. Möjligen om man använder kunskapen till destruktiva tillämpningar, t.ex. vapen. Men då är det tillämpningarna som är destruktiva, inte kunskapen. Sedan kan man även hävda att frontilinjeforskning inom vissa områden (t.ex. partikelfysik, astrofysik) är dyr eftersom den kräver stora och avancerade apparater.

/*fa*



/Peter E

Nyckelord: fysik, nytta med [6]; standardmodellen [24];

*

Kraft-Rörelse [19097]

Fråga:
5a)Hur mycket starkare är den starka kärnkraften mellan kvarkarna jämfört med den elektromagnetiska?

b)Hur mycket starkare är den starka kärnkraften mellan de två protonerna än den elektromagnetiska bortstötningen?

c)Hur mycket starkare är den elektromagnetiska bortstötningen mellan de två protonerna än tyngdkraften mellan de?

Tack på förhand!!
/linda x, init college of sweden, göteborg

Svar:
Eftersom krafterna har varierande avståndsberoende och beror på massa, laddning mm, så är exakta värden inte meningsfulla, se fråga 3716
/Peter E

*

Partiklar [19172]

Fråga:
Vilka partiklar är stabila?
/Veckans fråga

Ursprunglig fråga:
Hej! Vilka partiklar är stabila? Elektronen är det och protonen. Men anses kvarkar stabila? En nerkvark övergår i uppkvark när en neutron sönderfaller, så nerkvarken är väl inte stabil? Och är neutriner stabila, eller anses de sönderfalla? (De är ju rätt energirika, så de borde kunna sönderfalla.)
/Thomas Å, Knivsta

Svar:
Nedanstående bild visar alla elementarpartiklar enligt standardmodellen. Partiklarna är vertikalt ordnade efter vilomassa.

För att en partikel skall sönderfalla måste ett antal villkor (baserade på bevaringslagar) uppfyllas:

1 Sluttillståndet måste innehålla minst två partiklar
2 Partiklarna i sluttillståndet måste ha en sammanlagd vilomassa som är mindre än den sönderfallande partikelns massa (bevarande av energin)
3 Totala laddningen måste bevaras
4 Antalet fermioner (partiklar med halvtaligt spinn) kan bara ändras med ett jämnt antal (bevarande av rörelsemängdsmoment)
5 Antalet kvarkar bevaras (bevarande av baryontal)

De partiklar som uppfyller dessa villkor är

1 masslösa kraftförmedlarpartiklar: foton och gluon
2 elektron
3 proton
4 neutrino

Listan kräver några kommentarer:

Protonen är ingen elementarpartikel eftersom den består av tre kvarkar. Kvarkar kan emellertid inte förekomma isolerade, varför protonen kan betraktas som elementarpartikel. Protonens stabilitet implicerar då att den lättaste kvarkarna (upp och ner) är stabila i protonkonfigurationen (upp, upp, ner). Neutronen (upp, ner, ner) är emellertid inte stabil om den är isolerad. Den b-sönderfaller till en proton, se fråga 17998 . Tillsammans med rätt antal protoner kan neutronen emellertid vara stabil i en atomkärna.

Lägg även märke till att de två lättaste kvarkarna har mycket mindre massa än protoner och neutroner med massan c:a 1 GeV. Nukleonerna består alltså av mycket mer än tre kvarkar, den största delen av massan kommer från gluoner (Quark#Mass ).

Neutrinon är alltså i princip stabil, men den oscillerar mellan olika aromer (elektron, myon, tau), se Neutrino_oscillation .

Se även Particle_decay

Bilden nedan är från http://profmattstrassler.files.wordpress.com/2011/08/sm_masses2.png



/Peter E

Nyckelord: standardmodellen [24]; kvark [12]; neutrino [19];

1 http://profmattstrassler.com/articles-and-posts/particle-physics-basics/why-do-particles-decay/most-particles-decay-why/
2 http://profmattstrassler.com/articles-and-posts/particle-physics-basics/why-do-particles-decay/most-particles-decay-yet-some-dont/

*

Partiklar [19253]

Fråga:
1.Hur kan vi ha olika "partiklar" med samma kvark beståndsdelar?

2. ρ0 sönderfaller med den starka kraften till π+ och π−, vilket lämnar en resonans med en bredd på 150 MeV energi. Hur dess livslängd beräknas?

3. Varför är det omöjligt att mäta livslängden för ett svagt sönderfall tillstånd genom att mäta dess sönderfall bredd?
/Sara L, Lunduniversitet, Malmö

Svar:
1 Mesoner består t.ex. av en kvark och en antikvark. Eftersom kvarkar har spinn 1/2 kan man koppla till S=0 och S=1 vilket är olika mesoner. Dessutom kan partiklarna ha ett rörelsemängdsmoment L, vilket ger flera olika mesoner, se Meson#Spin,_orbital_angular_momentum,_and_total_angular_momentum .

2 Man kan relatera vidden och medellivslängden med Heisenbergs obestämdhetsrelation, se länk 1:

G = /t

Kalkylatorn i länk 1 ger för vidden 150 MeV livslängden 4*10-24 s.

3 Normalt är svaga sönderfall ganska långlivade vilket gör vidden liten. Om den naturliga vidden är mindre än den experimentella energiupplösningen ger vidden ingen information om livslängden.
/Peter E

Nyckelord: Heisenbergs obestämdhetsrelation [12];

1 http://hyperphysics.phy-astr.gsu.edu/hbase/quantum/parlif.html

*

Partiklar [19254]

Fråga:
Vad händer när en proton och en antiproton annihileras?
/Veckans fråga

Ursprunglig fråga:
1. En elektron och dess antipartikel, positron, förintas om de träffar på varandra. Det lär gälla även en proton och dess antipartikel, antiprotonen. Men hur kan då kvarkar och antikvarkar hålla sams? Mesoner sönderfaller men de annihilerar inte.

2.Skulle en "deuteriumkärna" kunna bestå av en proton och en antineutron? De är ju inte varandras antipartiklar.
/Thomas Å, Knivsta

Svar:
Hej Thomas!

1 Protonen och antiprotonen är ju sammansatta av kvarkar resp. antikvarkar. Till skillnad från elektron/positron så annihileras inte hela partikeln utan bara ett kvark-antikvark par, se figuren nedan från länk 2. Se även fråga 15922 .

Från Annihilation#Proton-antiproton_annihilation :

When a proton encounters its antiparticle (and more generally, if any species of baryon encounters any species of antibaryon), the reaction is not as simple as electron-positron annihilation. Unlike an electron, a proton is a composite particle consisting of three "valence quarks" and an indeterminate number of "sea quarks" bound by gluons. Thus, when a proton encounters an antiproton, one of its constituent valence quarks may annihilate with an antiquark, while the remaining quarks and antiquarks will undergo rearrangement into a number of mesons (mostly pions and kaons), which will fly away from the annihilation point. The newly created mesons are unstable, and will decay in a series of reactions that ultimately produce nothing but gamma rays, electrons, positrons, and neutrinos. This type of reaction will occur between any baryon (particle consisting of three quarks) and any antibaryon (consisting of three antiquarks). Antiprotons can and do annihilate with neutrons, and likewise antineutrons can annihilate with protons.

2 Som synes ovan är svaret nej. En proton och en antineutron annihileras på samma sätt som proton/antiproton.



/Peter E

Nyckelord: annihilation [14]; antimateria [16]; kvark [12];

*

Universum-Solen-Planeterna [19281]

Fråga:
Hej Fysikum I Big Bangmodellen förutsätts en tidsföljd, ex 10^-6 sek efter BB bildas av kvarkar protoner o neutroner. 1 sek efter BB kan protoner o neutroner bilda deuterium. Men i dubbelspaltexperimentet kan en störning påverka vad som borde ha skett innan störningen. En foton verkar befinna sig samtidigt, överallt. Detta motsäger väl tidsföljdsresonemanget i BB-modellen?
/Kaj A, Malmö

Svar:
Du har nog missuppfattat en del. Dubbelspaltexperimentet (fråga 5179 ) säger inget om tidsföljden. Den kvanteffekt som kommer in i big bang modellen är kvantfluktuationer som ger upphov till små temperaturskillnader i olika punkter, se fråga 11987 .
/Peter E

*

Partiklar [19597]

Fråga:
Jag är mycket fascinerad av vårt microuniversum men undrar om det är definitivt uträknat att vi har kommit till de minsta delarna? Mig veterligen så har vi i alla fall inte hittat den fysiska begränsningen på hur stort något kan vara och det innebär väl således att man inte heller kan begränsa hur smått något kan vara. Tankeexpriment "babushkadocka": Jag tänker om vår rymd så långt som vi känner till den är en del i ett större universum som i sin tur är en del i en större universum som i sin tur befolkas av intelligent liv så skulle det ta ganska lång tid för vetenskapen i deras universum att hitta ner till oss; och då har dem ändå en bit kvar för att hitta ner till det som vi hos oss anser vara det minste delarna i vårt universum. Det känns som att nuvarande teorier bygger på att det finns ett exakt värde för hur litet något kan vara men jag anser att det endast kan existera om det finns en exakt värde för hur stort något kan vara.
/Willie L, Säveskolan, Visby

Svar:
De flesta fysiker tror nog att kvarkar och leptoner är de minsta beståndsdelarna. Om de sedan kan beskrivas som svängande strängar och membran är än så länge spekulation.

Om man definierar fysiken som en beskrivning av man kan observera, så är kvarkar/leptoner de minsta enheterna. Sedan kan naturligtvis teoretiker spekulera om hur det skulle kunna vara.
/Peter E

*

Materiens innersta-Atomer-Kärnor [19656]

Fråga:
Hej! Vi håller på med kärnfysik på NOn just nu och jag hade en fråga angående betapartiklar som bildas i atomen då radioaktivt sönderfall sker. I min bok står det kort att "Man får tänka sig att en av neutronerna förvandlas till en proton och en elektron". Men jag har lite svårt att acceptera det svaret. Jag undrar vad det är som händer under själva processen när elektronen/protonen uppstår och hur en neutral partikel kan ge upphov till två laddade? Jag söker helt enkelt en lite mer ingående förklaring än den i vår fysikbok... Tacksam för svar!
/Mirjam S, Vallhallaskolan, Oskarshamn

Svar:
Så länge den totala laddningen bevaras är det inget problem med att en neutral partikel sönderfaller till laddade. Men om man tittar djupare (se nedan) är det faktiskt laddade partiklar (u- och d-kvarkar) involverade.

Se fråga 13471 för allmänt om betasönderfall. I fråga 12985 beskrivs processen i termer av den moderna bilden (standardmodellen, se fråga 18849 ) av den svaga växelverkan.
/Peter E

*

Partiklar [19966]

Fråga:
Annihilation
/Veckans fråga

Ursprunglig fråga:
Om materia och antimateria träffar varandra bildas bara fotoner. Innebär detta att den minsta beståndsdelen i universum (all materia) är fotoner ? Då blir diskussionen om andra minsta beståndsdelar onödig eller ? Tex om det är kvarkar, strängar mm ? Allt består i grunden av fotoner ?
/Göran A, Kungsbacka

Svar:
Annihilation uppstår när en partikel möter en antipartikel, och materia transformeras till energi i någon form.

Annihilation avser processer där en subatomär partikel kolliderar med sin antipartikel och förintas. Den totala energin som frigörs då (den massekvivalenta energin plus partiklarnas rörelseenergi) omvandlas direkt till elektromagnetisk strålning (QED) och i vissa fall till nya subatomära partiklar (QCD). Partikeln och dess antipartikel har exakt motsatta kvanttal och deras summa försvinner, så att också den resulterande skurens nya partiklar har i sin helhet kvanttal som är lika med noll.

Sluttillståndet kan alltså förutom fotoner även innehålla t.ex. kraftförmedlingspartiklarna gluoner eller W/Z.

Nej, mörk materia kan inte vara fotoner eftersom dessa växelverkar med materia genom partiklarnas laddning. Själva definitionen av mörk materia är ju att den inte växelverkar med materia på annat sätt än genom gravitationen.

Se även fråga 12396 19254 och mörk materia .
/Peter E

Nyckelord: annihilation [14]; mörk materia [17]; QED [7];

*

Materiens innersta-Atomer-Kärnor [20007]

Fråga:
Hej, kan du berätta vad hadronpartiklar är och vad man kan hitta de?
/Angelica D, Viktor Rydberg skola, Stockholm

Svar:
Hadroner är partiklar som är uppbyggda av kvarkar. Det finns två sorts hadroner: baryoner och mesoner. Se Hadron . Baryoner finns i protoner och neutroner. Mesoner bildas när baryoner kolliderar vid hög energi, t.ex. när kosmisk strålning träffar jordens atmosfär.
/Peter E

*

Materiens innersta-Atomer-Kärnor [20009]

Fråga:
Till vilken partikel kan man bryta ner materia och hur i så fall?
/Angelica D, Viktor Rydberg skola, Stockholm

Svar:
De minsta partiklarna är leptoner och kvarkar. Leptoner bildas vid betasönderfall. Kvarkar finns i protoner och neutroner, men kan inte förekomma fria.

Se vidare fråga 18849 .
/Peter E

*

Partiklar [20015]

Fråga:
Higgspartikeln
/Veckans fråga

Ursprunglig fråga:
Hej! Elektriska fält, magnetiska fält och gravitationsfält är bekanta och påvisbara. Det s k Higgsfältet har tillkommit. Hur påvisar man det? Finns områden där det saknas eller är starkare?
/Thomas Å, Knivsta

Svar:
Higgsbosonen (även: Higgs boson eller Higgspartikeln) är en partikel i partikelfysikens standardmodell, som genom Higgsmekanismen och Higgs-fältet beskriver varför partiklar har massa. Vid ett seminarium vid CERN i början av juli 2012 tillkännagav talespersonen Fabiola Gianotti att man sannolikt upptäckt Higgsbosonen. (Higgsboson )

Att elementarpartiklar får massa genom växelverkan med Higgsfältet kan liknas vid att fältet utsätter partiklarna för friktion. Higgsfältet genomsyrar vakuum. Ett vakuum som rubbas ger upphov till en vågrörelse hos Higgsfältet och "vågtopparna" är det som kallas för Higgspartikeln eller Higgsbosonen. Den skalära Higgsbosonen förmedlar växelverkan mellan Higgsfältet och elementarpartiklar, som utgörs av kvarkar och leptoner och bosoner. Att olika partiklar har olika massa har populärt beskrivits som djup snö, där vissa partiklar vadar i motstånd, medan andra kan glida ovanpå. (Wikipedia Higgsmekanismen )

Higgsfältet är inte som andra fält i kvantfysiken. De andra, som gravitation, varierar i styrka, och när de hamnar i sin lägsta energinivå antar de värdet noll. Så fungerar inte Higgsfältet. Till och med om rymden töms på allt och bara vakuum blir kvar, så är detta vakuum fortfarande fyllt med en substans, ett spöklikt fält som vägrar att stängas av – Higgsfältet. Vi märker det inte – Higgs­fältet är som luft för oss, som vatten för fiskarna. Men utan det skulle vi inte finnas, för det är genom att doppas i Higgsfältet som partiklarna får sin massa. De partiklar som far igenom utan att märka Higgsfältet får ingen massa, de som växelverkar svagt blir lätta och några blir, när de segar sig igenom fältet, riktigt tunga. (länk 1)

Frågan är, Thomas, hur mycket klokare man blir av det . Ett problem är att massa uppenbarligen kopplas till gravitation medan gravitationen inte finns med i standardmodellen.

Här är en föreläsning av Leonard Susskind, Stanford University om standardmodellen och hur higgspartikeln ger elementarpartiklarna massa.

Länk 2 är nobelkommitténs som vanligt utmärkta populära beskrivning av standardmodellen och higgspartikelns betydelse.
/Peter E

Nyckelord: standardmodellen [24]; higgspartikeln [10]; nyheter [11];

1 http://fof.se/tidning/2013/6/artikel/i-morkret-bortom-higgs
2 https://www.nobelprize.org/nobel_prizes/physics/laureates/2013/popular.html

*

[20020]

Fråga:
Hej! Higgsmekanismen ger materia massa. Hur får då energi massa? Potentiell energi i en spänd fjäder gör att fjäderns massa ökar. Större delen av en protons massa är inte kvarkar(materia) utan energi hos de masslösa gluonerna. Hur får energin massa?
/Thomas Å, Knivsta

Svaret kommer snart...

*

Universum-Solen-Planeterna [20517]

Fråga:
1. Roterar allt i universum åt samma håll? 2. En ide- Big bang, var ett svart hål från början som sög in materia och vars rotationsenergi överträffade gravitationen så att kvarkar och allt, som inte kan existera i fritt tillstånd gav upphov till explosionen och inflationen. Och detta kan ske igen i ett nytt svart hål någon annanstans. Vad tror ni om detta? Tack/ Joakim
/Joakim M, Kungsbacka

Svar:
Nej, allt roterar inte åt ett håll. Mätningar av rotationen hos ett stort antal galaxer ger dock en asymmetri på 7%. Detta anses emellertid inte helt övertygande.

Den kosmologiska principen
Det grundläggande antagandet inom kosmologin är att universum är homogent och isotropiskt, dvs det ser ungefär likadant ut, oavsett var man än befinner sig och vart man än tittar, se Kosmologiska_principen .

Länk 1 är ett senare arbete där man gjort mycket detaljerade mätningar på den kosmiska bakgrundsstrålningen (se fråga 705 ). Detta är ett citat från huvudförfattaren till artikeln:

"You can never rule it out completely, but we now calculate the odds that the universe prefers one direction over another at just 1 in 121,000. We're very glad that our work vindicates what most cosmologists assume. For now, cosmology is safe."

Ett roterande universum skulle dessutom vara svårt förstå om man accepterar inflationsteorin (se fråga 17472 ) eftersom den snabba expansionen borde sudda ut en rotation.
/Peter E

Nyckelord: kosmologi [33]; kosmisk bakgrundsstrålning [19];

1 https://phys.org/news/2016-09-scientists-universe.html

*

Partiklar [20591]

Fråga:
Om kvarkarna i en proton väger 12 MeV, hur kan protonen väga 938 MeV?
/Veckans fråga

Ursprunglig fråga:
Kvarkarna som hålls ihop av gluoner påstås vara masslösa, hur kan de då komma sig att en proton väger 938 MeV medan men totala massan av kvarkarna den består av endast väger 12 MeV?
/Emil A, Rudbeck, Örebro

Svar:
Ja, det kan tyckas konstigt. Problemet är att protonen inte bara består av tre kvarkar. Den innehåller (se nedanstående figur från länk 1) även massor av gluoner. Gluonerna har färger, så de kan växelverka med varandra och bilda virtuella kvark-antikvark par. I figuren är de ensamma blå bollarna (markerade) de tre kvarkarna. Övriga bollar är kvark-antikvark par, och spiralerna är gluoner. I själva verket kommer huvuddelen av protonens massa från gluonerna och de virtuella kvark-antikvark paren.

Detta är liknande fenomentet i QED (se nedan) att vakuum har en energi som kommer från virtuella elektron-positron par, se fråga 11001 och Vacuum_energy .

Se även Proton#Quarks_and_the_mass_of_a_proton .

Kvantteorin för den elektromagnetiska växelverkan är QED och motsvarande teori för färgkraften (stark växelverkan) är QCD. Här är lite av vad Wikipedia säger om dessa:

QED
Kvantelektrodynamik (QED efter engelska Quantum electrodynamics) är en fysikalisk teori grundad på kvantfysik och elektrodynamik som kan sägas vara en tillämpning av kvantfältteori på elektromagnetiska fält.

Under 1940-talet hade Feynman, Schwinger och Tomonaga var för sig lyckats visa att elektromagnetismen kunde skrivas som en fullgod kvantteori. Problemet var att enligt den relativistiska kvantmekaniken kan partiklar skapas om man har tillräckligt med energi. Detta betyder att då man sprider en elektron mot en annan elektron kan man skapa t.ex. ett extra elektron-positronpar. Har man inte tillräckligt med energi kan man ändå skapa dem virtuellt, eftersom Heisenbergs osäkerhetsprincip säger att så länge detta par lever tillräcklig kort tid kan det skapas. Detta betyder att man måste behandla teorin som en mångpartikelteori, en kvantfältteori, där man kan skapa ett godtyckligt antal partiklar så länge detta inte strider mot osäkerhetsprincipen. (Kvantelektrodynamik )

QCD
Kvantkromodynamik eller QCD (från eng. quantum chromodynamics) är inom partikelfysiken den teoretiska beskrivningen av stark växelverkan. Den starka kraften binder samman kvarkar till protoner, neutroner och andra hadroner och den håller också samman atomkärnorna som dessa protoner och neutroner bygger upp. Kvantkromodynamiken är en kvantfältteori som på den fundamentala nivån beskriver hur kvarkar växelverkar genom att byta ut masslösa partiklar med spinn 1 som kallas gluoner. (Kvantkromodynamik )

Se även fråga 20647 .



/Peter E

Nyckelord: vakuum [9]; kvark [12]; gluoner [7]; QED [7];

1 http://cosmologyscience.com/cosblog/three-quarks-dont-add-up-to-one-proton-not-even-close/

*

Partiklar [20647]

Fråga:
Hej! Som jag förstått det kommer den allra största delen av en protons massa från energin som håller ihop partikeln och inte från någon "materia". Energin har massa. Hur stor del av protonen består av energi och hur stor är "resten" och vad består den av? Är mängden energi som ger massa likadan i andra partiklar, t ex en meson, elektron, neutrino etc?
/Thomas Å, Knivsta

Svar:
Detta är en besvärlig fråga eftersom fria kvarkar inte existerar och kan därför inte tilldelas någon massa. Vi har behandlat problemet i fråga 20591 . Det diskuteras även i länk 1.

Om vi i stället tittar på en atomkärna som ju är sammansatt av nukleoner (neutroner och protoner). Om vi sätter samman en atomkärna med N neutroner och Z protoner, så finner vi att den sammansatta kärnan har mindre massa än de fria beståndsdelarna. Massan (energin) som fattas har strålat bort när kärnan sattes samman. Man kan se det så att den attraktiva bindningen i atomkärnan skapar en negativ potential som stänger inne nukleonerna i kärnan.

Vad gäller kvarkarnas massor så ser man i nedanstående figur (från länk 1) att protonen har mycket större massa än summan av de ingående kvarkarna. Dessutom är kvarkmassorna modellberoende, så värdena kan variera, se länk 1,2, Quark#Mass och Proton#Quarks_and_the_mass_of_a_proton .

Mesoner är sammansatta av en kvark och en antikvark, så förhållandet är samma som för protonen att en stor del av massan kommer från kvark/antikvark par och gluonerna.

Vad gäller elektronen och neutrinen så är de elementarpartiklar (ej uppbyggda av andra partiklar, så de får sin massa direkt av higgsmekanismen, se fråga 20015 .



/Peter E

Nyckelord: kvark [12]; bindningsenergi [23];

1 https://physics.stackexchange.com/questions/185149/where-does-the-majority-of-the-mass-of-the-usual-matter-come-from
2 http://hyperphysics.phy-astr.gsu.edu/hbase/Particles/quark.html#c1

*

Universum-Solen-Planeterna [20668]

Fråga:
Hej! Man anser sig kunna räkna bakåt till hur universum "såg ut"/"fungerade" strax efter Stora Smällen. När började regler som Pauliprincipen att gälla? Alldeles vid Smällen lär den ju inte ha gällt, och en följdfråga är: Vad som krävs för att våra fysikprinciper skall sluta att gälla, t ex i svarta hål eller andra "svåra platser". Finns några "regler" kvar?
/Thomas Å, Knivsta

Svar:
Hej Thomas! Det enkla svaret är att man inte vet. Det som är det älsta observerade är ju den kosmiska bakgrundsstrålningen (se fråga 705 ) från c:a 380000 år efter big bang. Allt tidigare än detta är teoretiska extrapolationer. Möjligen kan de nyligen upptäckta gravitationsvågorna (se fråga 20117 ) tillåta att man kan "se" igenom vallen av bakgrundsstrålning.

Vad gäller pauliprincipen (se fråga 18298 ) så tror jag inte det är något problem. Min föreställning är att fermioner (kvarkar/leptoner) inte bidades från den heta strålningen förrän Pauli så tillät.

Enligt "inget-hår teoremet" skall man inte behöva oroa sig för pauliprincipen i ett svart hål. Det enda man kan veta är hålets massa, laddning och rörelsemängdsmoment (se No-hair_theorem ).
/Peter E

Nyckelord: big bang [37]; svart hål [51];

*

Materiens innersta-Atomer-Kärnor [20887]

Fråga:
Då ett ämne sönderfaller och skickar ut betastrålning så ökar atomnumret med ett. En neutron blir en proton. Men var kommer elektronen från? Är det så att differensen mellan +2/3 och -1/3 och utbyte av dessa blir bettsönderfallet?
/Lasse G, Sophiaskolan, Rörum

Svar:
Ja, i grunden är betasönderfallet övergångar mellan u (+2/3) och d (-1/3) kvarkar som finns i neutroner och protoner i kärnan, se fråga 12985 .
/Peter E

*

Partiklar [20892]

Fråga:
Hej! Kan en elektron delas? Den kan ju uppkomma när kvarkar omvandlas vid neutronsönderfall. Kan reaktionen gå åt andra hållet? (I skolan sägs det ofta att elektronen är odelbar.)
/Thomas Å, Knivsta

Svar:
Nej, elektronen kan inte delas, därför kallas den elementarpartikel.

Alla grundläggande fysikaliska processer, som t.ex. betasönderfallet i fråga 12985 , är i princip reversibla. Sannolikheten för en reaktion åt andra hållet kan emellertid vara mycket liten. Se Irreversible_process#Absolute_versus_statistical_reversibility .
/Peter E

*

Partiklar [20910]

Fråga:
Hej! Anti-neutriner bildas vid betasönderfall. Dessa antineutriner bör väl ganska snabbt förintas genom att de träffar på neutriner och ge upphov till mätbar strålning (i mikrovågsområdet). Är sådan strålning påvisad? Ger den någon antydan om neutrinernas egenskaper?
/Thomas Å, Knivsta

Svar:
Sannolikheten att neutrinerna skall kollidera och annihilera varandra är mycket liten. Om de gör det händer följande enligt länk 1: Neutrinerna annihilerar varandra och bildar en virtuell Z bozon, vilken är den neutrala bäraren av den svaga kraften. Z-bozonen sönderfaller omedelbart till ett annat partikel/antipartikel par - t.ex. neutriner, leptoner (elektroner/positroner) eller kvarkar. Vad som bildas beror på hur mycket energi de kolliderande neutrinerna har.

Eftersom man inte kan producera koncentrerade strålar av neutriner har neutrino/antineutrino annihilation inte observerats. Möjligen kan de förekomma i aktiva svarta hål.

neutrino innehåller massor av information om neutriner.
/Peter E

Nyckelord: neutrino [19];

1 http://www.madsci.org/posts/archives/2006-10/1161025690.Ph.r.html

*

Materiens innersta-Atomer-Kärnor [21231]

Fråga:
Vad är laddning hos en elementarpartikel? Vilka oladdade finns det?
/Veckans fråga

Ursprunglig fråga:
Hej! Jag undrar lite om elementarpartiklar. En elementarpartikel har alltid en laddning, men vad är laddningen egentligen? Är det att energin/massan är positiv/negativ eller är det bara något "magiskt" som händer med partikeln? Samt hur kan jag se på en partikel vilken laddning den har?
/Elin E, Lund

Svar:
Elementarpartiklar är materiens minsta beståndsdelar. Partiklar som har mindre beståndsdelar räknas inte som elementarpartiklar. Till elementarpartiklarna räknar man också de partiklar som är bärare av de fyra fundamentala krafterna i naturen. Elementarpartiklar studeras inom partikelfysiken, där de partiklar man för närvarande känner till beskrivs av den så kallade Standardmodellen. (Elementarpartikel , Standardmodellen )

Det finns ett antal elementarpartiklar som är oladdade, nämligen tre neutriner, fotonen och Z-bozonen. Vad gäller övriga partiklar så har kvarkar +-(1/3)e laddningar och laddade leptoner laddningen +-e.

Laddade partiklar påverkar varandra med den elektromagnetiska kraften. I artikeln Elektrisk_laddning beskrivs vad laddning är.

Vilken laddning en partikel har bestäms enkelt genom att låta partikeln avlänkas med ett magnetfält med känd riktning.
/Peter E

Nyckelord: standardmodellen [24];

*

Materiens innersta-Atomer-Kärnor [21232]

Fråga:
Hej! Eftersom en foton är masslös rör den sig alltid i ljusets hastighet. Gäller detta även gluonen som också är masslös?
/Elin E, Lund

Svar:
Ja, gluonen (se Gluon och fråga 1720 ) saknar vilomassa och rör sig därför med ljushastigheten Massless_particle#Special_relativity . Gluonen är emellertid instängd genom den starka färgkraften, så den har inte mycket frihet att färdas med ljushastigheten.

Innestängningen kommer sig av att färgkraften blir starkare ju större avståndet mellan gluoner/kvarkar är (i stället för att avta med avståndet som den elektromagnetiska kraften). Om man försöker separera dem genom att tillföra energi bildas i stället nya gluoner eller kvark/antikvark par, se fråga 3931 .
/Peter E

Nyckelord: gluoner [7];

*

Universum-Solen-Planeterna [21417]

Fråga:
När kvarkar och leptoner bildades efter big bang, varför fick dom de egenskaper de har ? En del säger att det var slumpmässigt, men ändå så blev tex alla elektroner lika, och alla kvarkar av samma typ lika ? Universum var väl ändå hyfsat stort när de bildades. Om det hade varit slumpen så borde det blivit en stor mängd med olika sorters kvarkar resp elektroner med olika egenskaper, men nu är alla exakt lika -- ? Det måste finnas någon underliggande orsak. Finns det någon teori om detta ?

Samma fråga när det gäller tex kortlivade & instabila kvarkar som enbart bildas i kollisioner, inget slumpmässigt där inte, utan egenskaperna blir alltid samma ?
/Hans Å, Stockholm

Svar:
Jag brukar säga att i fysiken kan man aldrig svara på "varför" utan man får nöja sig med "hur".

Den antropiska principen är, inom kosmologi, fysik och filosofi, ett begrepp som sammanfattar den vetenskapliga insikten att universums grundläggande naturlagar måste ha en specifik utformning för att liv ska kunna uppstå. Till exempel skulle en obetydlig ökning av utgångshastigheten vid universums begynnelse ha omöjliggjort för tyngdlagen att dra samman lokala öar av materia för att bilda galaxer och stjärnor. Om den starka kärnkraften hade varit enbart en aning starkare än den är skulle det varken finnas vatten eller tillräckligt stabila stjärnor för att liv skulle kunna utvecklas på jorden. (Antropiska_principen )

Så för att universum skall kunna utveckla liv krävs att naturlagarna utformas på ett visst sätt. Antingen finns vi då till av en osannolik slump eller så finns det oändligt många multiversa.

Multiversum, eller "mångvärldshypotesen", är ett begrepp som beskriver en världsbild med många eller oändligt många universum. Det kan också syfta på ett kosmos där det finns ett oändligt antal parallella universum. Teorin om multiversum är ett svar på den s.k. antropiska principen. (Multiversum )

Endast en liten bråkdel av universa har kunnat utveckla intelligent (?) liv. För de övriga finns ingen som kan fundera på varför de inte finns.

För en lättillgänglig framställning av kosmologin (se fråga 18978 ) kan jag rekommendera Ulf Danielssons bok Mörkret vid tidens ände, se länk 1.
/Peter E

Nyckelord: kosmologi [33];

1 http://tidskriftenrespons.se/recension/iden-om-att-det-finns-andra-universum-gar-inte-att-testa/

*

Materiens innersta-Atomer-Kärnor [21434]

Fråga:
Hej! Vi läser kärnfysik i skolan, och jag undrar hur en neutron kan falla sönder till en proton, en elektron och en antineutrino. Både neutroner och protoner består ju av kvarkar med olika laddningar, men det gör ju inte en elektron, och en antineutrino är ju knappt en massa. Hur går ett betasönderfall till, om man ska förklara det på en högre nivå?
/Gunnel G, Kungsholmens Grundskola, Stockholm

Svar:
I kvarkmodellen för betasönderfall förvandlas en u-kvark till en d-kvark eller tvärtom under utsändning av en laddad lepton (elektron eller positron) och en oladdad lepton (antineutrino eller neutrino), se fråga 12985 .
/Peter E

Nyckelord: betasönderfall [15];

*

Sök efter    

Skriv de ord du vill söka på i sökfältet ovan och klicka på sökknappen. Uteslut ord genom att sätta - (minus) före ordet. Ordgrupper definieras med hjälp av "...". Sökningar är oberoende av stora och små bokstäver.

Exempel:

helium "kalle anka"
Sök på 'helium' och ordgruppen 'kalle anka'
orgelpipa
Sök på 'orgelpipa'
orgel -gitarr
Sök på 'orgel' men inte 'gitarr'

 


sök | söktips | Veckans fråga | alla 'Veckans fråga' | ämnen | dokumentation | ställ en fråga
till diskussionsfora

 

Creative Commons License

Denna sida från NRCF är licensierad under Creative Commons:
Erkännande-Ickekommersiell-Inga bearbetningar
.